Programs & Examples On #External script

Android view pager with page indicator

Just an improvement to the nice answer given by @vuhung3990. I implemented the solution and works great but if I touch one radio button it will be selected and nothing happens.

I suggest to also change page when a radio button is tapped. To do this, simply add a listener to the radioGroup:

mPager = (ViewPager) findViewById(R.id.pager);
final RadioGroup radioGroup = (RadioGroup)findViewById(R.id.radiogroup);    
radioGroup.setOnCheckedChangeListener(new RadioGroup.OnCheckedChangeListener() {
            @Override
            public void onCheckedChanged(RadioGroup group, int checkedId) {
                switch (checkedId) {
                    case R.id.radioButton :
                        mPager.setCurrentItem(0, true);
                        break;
                    case R.id.radioButton2 :
                        mPager.setCurrentItem(1, true);
                        break;
                    case R.id.radioButton3 :
                        mPager.setCurrentItem(2, true);
                        break;
                }
            }
        });

Loop through files in a folder in matlab

Looping through all the files in the folder is relatively easy:

files = dir('*.csv');
for file = files'
    csv = load(file.name);
    % Do some stuff
end

Powershell: A positional parameter cannot be found that accepts argument "xxx"

I had this problem when trying to change directory, using the character _. The solution was to use a string to change directories.

C:\> cd "my_new_dir"

Android Design Support Library expandable Floating Action Button(FAB) menu

When I tried to create something simillar to inbox floating action button i thought about creating own custom component.

It would be simple frame layout with fixed height (to contain expanded menu) containing FAB button and 3 more placed under the FAB. when you click on FAB you just simply animate other buttons to translate up from under the FAB.

There are some libraries which do that (for example https://github.com/futuresimple/android-floating-action-button), but it's always more fun if you create it by yourself :)

How do I include a file over 2 directories back?

../ is one directory, Repeat for two directories ../../ or even three: ../../../ and so on.

Defining constants may reduce confusion because you will drill forward into directories verses backwards

You could define some constants like so:

define('BD', '/home/user/public_html/example/');

define('HTMLBD', 'http://example.com/');

When using 'BD' or my 'base directory' it looks like so:

file(BD.'location/of/file.php');

define(); reference

C# Double - ToString() formatting with two decimal places but no rounding

I know this is a old thread but I've just had to do this. While the other approaches here work, I wanted an easy way to be able to affect a lot of calls to string.format. So adding the Math.Truncate to all the calls to wasn't really a good option. Also as some of the formatting is stored in a database, it made it even worse.

Thus, I made a custom format provider which would allow me to add truncation to the formatting string, eg:

string.format(new FormatProvider(), "{0:T}", 1.1299); // 1.12
string.format(new FormatProvider(), "{0:T(3)", 1.12399); // 1.123
string.format(new FormatProvider(), "{0:T(1)0,000.0", 1000.9999); // 1,000.9

The implementation is pretty simple and is easily extendible to other requirements.

public class FormatProvider : IFormatProvider, ICustomFormatter
{
    public object GetFormat(Type formatType)
    {
        if (formatType == typeof (ICustomFormatter))
        {
            return this;
        }
        return null;
    }

    public string Format(string format, object arg, IFormatProvider formatProvider)
    {
        if (arg == null || arg.GetType() != typeof (double))
        {
            try
            {
                return HandleOtherFormats(format, arg);
            }
            catch (FormatException e)
            {
                throw new FormatException(string.Format("The format of '{0}' is invalid.", format));
            }
        }

        if (format.StartsWith("T"))
        {
            int dp = 2;
            int idx = 1;
            if (format.Length > 1)
            {
                if (format[1] == '(')
                {
                    int closeIdx = format.IndexOf(')');
                    if (closeIdx > 0)
                    {
                        if (int.TryParse(format.Substring(2, closeIdx - 2), out dp))
                        {
                            idx = closeIdx + 1;
                        }
                    }
                    else
                    {
                        throw new FormatException(string.Format("The format of '{0}' is invalid.", format));
                    }
                }
            }
            double mult = Math.Pow(10, dp);
            arg = Math.Truncate((double)arg * mult) / mult;
            format = format.Substring(idx);
        }

        try
        {
            return HandleOtherFormats(format, arg);
        }
        catch (FormatException e)
        {
            throw new FormatException(string.Format("The format of '{0}' is invalid.", format));
        }
    }

    private string HandleOtherFormats(string format, object arg)
    {
        if (arg is IFormattable)
        {
            return ((IFormattable) arg).ToString(format, CultureInfo.CurrentCulture);
        }
        return arg != null ? arg.ToString() : String.Empty;
    }
}

Generating UML from C++ code?

Whoever wants UML deserves Rational Rose :)

HTTP Ajax Request via HTTPS Page

In some cases a one-way request without a response can be fired to a TCP server, without a SSL certificate. A TCP server, in contrast to a HTTP server, will catch you request. However there will be no access to any data sent from the browser, because the browser will not send any data without a positive certificate check. And in special cases even a bare TCP signal without any data is enough to execute some tasks. For example for an IoT device within a LAN to start a connection to an external service. Link

This is a kind of a "Wake Up" trigger, that works on a port without any security.

In case a response is needed, this can be implemented using a secured public https server, which can send the needed data back to the browser using e.g. Websockets.

Python - List of unique dictionaries

Well all the answers mentioned here are good, but in some answers one can face error if the dictionary items have nested list or dictionary, so I propose simple answer

a = [str(i) for i in a]
a = list(set(a))
a = [eval(i) for i in a]

How do I apply a perspective transform to a UIView?

As Ben said, you'll need to work with the UIView's layer, using a CATransform3D to perform the layer's rotation. The trick to get perspective working, as described here, is to directly access one of the matrix cells of the CATransform3D (m34). Matrix math has never been my thing, so I can't explain exactly why this works, but it does. You'll need to set this value to a negative fraction for your initial transform, then apply your layer rotation transforms to that. You should also be able to do the following:

Objective-C

UIView *myView = [[self subviews] objectAtIndex:0];
CALayer *layer = myView.layer;
CATransform3D rotationAndPerspectiveTransform = CATransform3DIdentity;
rotationAndPerspectiveTransform.m34 = 1.0 / -500;
rotationAndPerspectiveTransform = CATransform3DRotate(rotationAndPerspectiveTransform, 45.0f * M_PI / 180.0f, 0.0f, 1.0f, 0.0f);
layer.transform = rotationAndPerspectiveTransform;

Swift 5.0

if let myView = self.subviews.first {
    let layer = myView.layer
    var rotationAndPerspectiveTransform = CATransform3DIdentity
    rotationAndPerspectiveTransform.m34 = 1.0 / -500
    rotationAndPerspectiveTransform = CATransform3DRotate(rotationAndPerspectiveTransform, 45.0 * .pi / 180.0, 0.0, 1.0, 0.0)
    layer.transform = rotationAndPerspectiveTransform
}

which rebuilds the layer transform from scratch for each rotation.

A full example of this (with code) can be found here, where I've implemented touch-based rotation and scaling on a couple of CALayers, based on an example by Bill Dudney. The newest version of the program, at the very bottom of the page, implements this kind of perspective operation. The code should be reasonably simple to read.

The sublayerTransform you refer to in your response is a transform that is applied to the sublayers of your UIView's CALayer. If you don't have any sublayers, don't worry about it. I use the sublayerTransform in my example simply because there are two CALayers contained within the one layer that I'm rotating.

How to create a Restful web service with input parameters?

You can. Try something like this:

@Path("/todo/{varX}/{varY}")
@Produces({"application/xml", "application/json"})
public Todo whatEverNameYouLike(@PathParam("varX") String varX,
    @PathParam("varY") String varY) {
        Todo todo = new Todo();
        todo.setSummary(varX);
        todo.setDescription(varY);
        return todo;
}

Then call your service with this URL;
http://localhost:8088/JerseyJAXB/rest/todo/summary/description

How to scroll to top of the page in AngularJS?

Ideally we should do it from either controller or directive as per applicable. Use $anchorScroll, $location as dependency injection.

Then call this two method as

$location.hash('scrollToDivID');
$anchorScroll();

Here scrollToDivID is the id where you want to scroll.

Assumed you want to navigate to a error message div as

<div id='scrollToDivID'>Your Error Message</div>

For more information please see this documentation

Eclipse: Error ".. overlaps the location of another project.." when trying to create new project

simply "CUT" project folder and move it out of workspace directory and do the following

file=>import=>(select new directory)=> mark (copy to my workspace) checkbox 

and you done !

How to clear a data grid view

Firstly, null the data source:

this.dataGridView.DataSource = null;

Then clear the rows:

this.dataGridView.Rows.Clear();

Then set the data source to the new list:

this.dataGridView.DataSource = this.GetNewValues();

What do .c and .h file extensions mean to C?

The .c files are source files which will be compiled. The .h files are used to expose the API of a program to either other part of that program or other program is you are creating a library.

For example, the program PizzaDelivery could have 1 .c file with the main program, and 1 .c file with utility functions. Now, for the main part of the program to be able to use the utility functions, you need to expose the API, via function prototype, into a .h file, this .h file being included by the main .c file.

how to change default python version?

On Mac OS X using the python.org installer as you apparently have, you need to invoke Python 3 with python3, not python. That is currently reserved for Python 2 versions. You could also use python3.2 to specifically invoke that version.

$ which python
/usr/bin/python
$ which python3
/Library/Frameworks/Python.framework/Versions/3.2/bin/python3
$ cd /Library/Frameworks/Python.framework/Versions/3.2/bin/
$ ls -l
total 384
lrwxr-xr-x  1 root  admin      8 Apr 28 15:51 2to3@ -> 2to3-3.2
-rwxrwxr-x  1 root  admin    140 Feb 20 11:14 2to3-3.2*
lrwxr-xr-x  1 root  admin      7 Apr 28 15:51 idle3@ -> idle3.2
-rwxrwxr-x  1 root  admin    138 Feb 20 11:14 idle3.2*
lrwxr-xr-x  1 root  admin      8 Apr 28 15:51 pydoc3@ -> pydoc3.2
-rwxrwxr-x  1 root  admin    123 Feb 20 11:14 pydoc3.2*
-rwxrwxr-x  2 root  admin  25624 Feb 20 11:14 python3*
lrwxr-xr-x  1 root  admin     12 Apr 28 15:51 python3-32@ -> python3.2-32
lrwxr-xr-x  1 root  admin     16 Apr 28 15:51 python3-config@ -> python3.2-config
-rwxrwxr-x  2 root  admin  25624 Feb 20 11:14 python3.2*
-rwxrwxr-x  1 root  admin  13964 Feb 20 11:14 python3.2-32*
lrwxr-xr-x  1 root  admin     17 Apr 28 15:51 python3.2-config@ -> python3.2m-config
-rwxrwxr-x  1 root  admin  25784 Feb 20 11:14 python3.2m*
-rwxrwxr-x  1 root  admin   1865 Feb 20 11:14 python3.2m-config*
lrwxr-xr-x  1 root  admin     10 Apr 28 15:51 pythonw3@ -> pythonw3.2
lrwxr-xr-x  1 root  admin     13 Apr 28 15:51 pythonw3-32@ -> pythonw3.2-32
-rwxrwxr-x  1 root  admin  25624 Feb 20 11:14 pythonw3.2*
-rwxrwxr-x  1 root  admin  13964 Feb 20 11:14 pythonw3.2-32*

If you also installed a Python 2 from python.org, it would have a similar framework bin directory with no overlapping file names (except for 2to3).

$ open /Applications/Python\ 2.7/Update\ Shell\ Profile.command
$ sh -l
$ echo $PATH
/Library/Frameworks/Python.framework/Versions/2.7/bin:/Library/Frameworks/Python.framework/Versions/3.2/bin:/usr/bin:/bin:/usr/sbin:/sbin:/usr/local/bin
$ which python3
/Library/Frameworks/Python.framework/Versions/3.2/bin/python3
$ which python
/Library/Frameworks/Python.framework/Versions/2.7/bin/python
$ cd /Library/Frameworks/Python.framework/Versions/2.7/bin
$ ls -l
total 288
-rwxrwxr-x  1 root  admin    150 Jul  3  2010 2to3*
lrwxr-x---  1 root  admin      7 Nov  8 23:14 idle@ -> idle2.7
-rwxrwxr-x  1 root  admin    138 Jul  3  2010 idle2.7*
lrwxr-x---  1 root  admin      8 Nov  8 23:14 pydoc@ -> pydoc2.7
-rwxrwxr-x  1 root  admin    123 Jul  3  2010 pydoc2.7*
lrwxr-x---  1 root  admin      9 Nov  8 23:14 python@ -> python2.7
lrwxr-x---  1 root  admin     16 Nov  8 23:14 python-config@ -> python2.7-config
-rwxrwxr-x  1 root  admin  33764 Jul  3  2010 python2.7*
-rwxrwxr-x  1 root  admin   1663 Jul  3  2010 python2.7-config*
lrwxr-x---  1 root  admin     10 Nov  8 23:14 pythonw@ -> pythonw2.7
-rwxrwxr-x  1 root  admin  33764 Jul  3  2010 pythonw2.7*
lrwxr-x---  1 root  admin     11 Nov  8 23:14 smtpd.py@ -> smtpd2.7.py
-rwxrwxr-x  1 root  admin  18272 Jul  3  2010 smtpd2.7.py*

Hibernate error: ids for this class must be manually assigned before calling save():

Resolved this problem using a Sequence ID defined in Oracle database.

ORACLE_DB_SEQ_ID is defined as a sequence for the table. Also look at the console to see the Hibernate SQL that is used to verify.

@Id
@Column(name = "MY_ID", unique = true, nullable = false)
@GeneratedValue(strategy=GenerationType.SEQUENCE, generator = "id_Sequence")
@SequenceGenerator(name = "id_Sequence", sequenceName = "ORACLE_DB_SEQ_ID")
Long myId;

Matching an empty input box using CSS

It worked for me to add a class name to the input and then apply CSS rules to that:

<input type="text" name="product" class="product" />

<style>
input[value=""].product {
    display: none;
}
</style>

How to restore the dump into your running mongodb

  1. Start mongod
  2. Navigate to folder where you have extracted "enron.zip" in OS shell(cmd in case of windows)
  3. Then type ">mongorestore -d your_db_name dump/enron"

Set HTTP header for one request

Try this, perhaps it works ;)

.factory('authInterceptor', function($location, $q, $window) {


return {
    request: function(config) {
      config.headers = config.headers || {};

      config.headers.Authorization = 'xxxx-xxxx';

      return config;
    }
  };
})

.config(function($httpProvider) {
  $httpProvider.interceptors.push('authInterceptor');
})

And make sure your back end works too, try this. I'm using RESTful CodeIgniter.

class App extends REST_Controller {
    var $authorization = null;

    public function __construct()
    {
        parent::__construct();
        header('Access-Control-Allow-Origin: *');
        header("Access-Control-Allow-Headers: X-API-KEY, Origin, X-Requested-With, Content-Type, Accept, Access-Control-Request-Method, Authorization");
        header("Access-Control-Allow-Methods: GET, POST, OPTIONS, PUT, DELETE");
        if ( "OPTIONS" === $_SERVER['REQUEST_METHOD'] ) {
            die();
        }

        if(!$this->input->get_request_header('Authorization')){
            $this->response(null, 400);    
        }

        $this->authorization = $this->input->get_request_header('Authorization');
    }

}

Delete last char of string

string.Join is better, but if you really want a LINQ ForEach:

var strgroupids = string.Empty;

groupIds.ForEach(g =>
{
    if(strgroupids != string.Empty){
        strgroupids += ",";
    }

    strgroupids += g;
});

Some notes:

  • string.Join and foreach are both better than this, vastly slower, approach
  • No need to remove the last , since it's never appended
  • The increment operator (+=) is handy for appending to strings
  • .ToString() is unnecessary as it is called automatically when concatenating non-strings
  • When handling large strings, StringBuilder should be considered instead of concatenating strings

Use grep --exclude/--include syntax to not grep through certain files

If you just want to skip binary files, I suggest you look at the -I (upper case i) option. It ignores binary files. I regularly use the following command:

grep -rI --exclude-dir="\.svn" "pattern" *

It searches recursively, ignores binary files, and doesn't look inside Subversion hidden folders, for whatever pattern I want. I have it aliased as "grepsvn" on my box at work.

Getting and removing the first character of a string

There is also str_sub from the stringr package

x <- 'hello stackoverflow'
str_sub(x, 2) # or
str_sub(x, 2, str_length(x))
[1] "ello stackoverflow"

What is PEP8's E128: continuation line under-indented for visual indent?

PEP-8 recommends you indent lines to the opening parentheses if you put anything on the first line, so it should either be indenting to the opening bracket:

urlpatterns = patterns('',
                       url(r'^$', listing, name='investment-listing'))

or not putting any arguments on the starting line, then indenting to a uniform level:

urlpatterns = patterns(
    '',
    url(r'^$', listing, name='investment-listing'),
)

urlpatterns = patterns(
    '', url(r'^$', listing, name='investment-listing'))

I suggest taking a read through PEP-8 - you can skim through a lot of it, and it's pretty easy to understand, unlike some of the more technical PEPs.

Using a batch to copy from network drive to C: or D: drive

Just do the following change

echo off
cls

echo Would you like to do a backup?

pause

copy "\\My_Servers_IP\Shared Drive\FolderName\*" C:\TEST_BACKUP_FOLDER

pause

How does spring.jpa.hibernate.ddl-auto property exactly work in Spring?

For the record, the spring.jpa.hibernate.ddl-auto property is Spring Data JPA specific and is their way to specify a value that will eventually be passed to Hibernate under the property it knows, hibernate.hbm2ddl.auto.

The values create, create-drop, validate, and update basically influence how the schema tool management will manipulate the database schema at startup.

For example, the update operation will query the JDBC driver's API to get the database metadata and then Hibernate compares the object model it creates based on reading your annotated classes or HBM XML mappings and will attempt to adjust the schema on-the-fly.

The update operation for example will attempt to add new columns, constraints, etc but will never remove a column or constraint that may have existed previously but no longer does as part of the object model from a prior run.

Typically in test case scenarios, you'll likely use create-drop so that you create your schema, your test case adds some mock data, you run your tests, and then during the test case cleanup, the schema objects are dropped, leaving an empty database.

In development, it's often common to see developers use update to automatically modify the schema to add new additions upon restart. But again understand, this does not remove a column or constraint that may exist from previous executions that is no longer necessary.

In production, it's often highly recommended you use none or simply don't specify this property. That is because it's common practice for DBAs to review migration scripts for database changes, particularly if your database is shared across multiple services and applications.

Adding class to element using Angular JS

Use the MV* Pattern

Based on the example you attached, It's better in angular to use the following tools:

  • ng-click - evaluates the expression when the element is clicked (Read More)
  • ng-class - place a class based on the a given boolean expression (Read More)

for example:

<button ng-click="enabled=true">Click Me!</button>

<div ng-class="{'alpha':enabled}"> 
    ...
</div>

This gives you an easy way to decouple your implementation. e.g. you don't have any dependency between the div and the button.

Read this to learn about the MV* Pattern

Is there a better way to run a command N times in bash?

If you are OK doing it periodically, you could run the following command to run it every 1 sec indefinitely. You can put other custom checks in place to run it n number of times.

watch -n 1 some_command

If you wish to have visual confirmation of changes, append --differences prior to the ls command.

According to the OSX man page, there's also

The --cumulative option makes highlighting "sticky", presenting a running display of all positions that have ever changed. The -t or --no-title option turns off the header showing the interval, command, and current time at the top of the display, as well as the following blank line.

Linux/Unix man page can be found here

At least one JAR was scanned for TLDs yet contained no TLDs

For anyone trying to get this working using the Sysdeo Eclipse Tomcat plugin, try the following steps (I used Sysdeo Tomcat Plugin 3.3.0, Eclipse Kepler, and Tomcat 7.0.53 to construct these steps):

  1. Window --> Preferences --> Expand the Tomcat node in the tree --> JVM Settings
  2. Under "Append to JVM Parameters", click the "Add" button.
  3. In the "New Tomcat JVM parameter" popup, enter -Djava.util.logging.config.file="{TOMCAT_HOME}\conf\logging.properties", where {TOMCAT_HOME} is the path to your Tomcat directory (example: C:\Tomcat\apache-tomcat-7.0.53\conf\logging.properties). Click OK.
  4. Under "Append to JVM Parameters", click the "Add" button again.
  5. In the "New Tomcat JVM parameter" popup, enter -Djava.util.logging.manager=org.apache.juli.ClassLoaderLogManager. Click OK.
  6. Click OK in the Preferences window.
  7. Make the adjustments to the {TOMCAT_HOME}\conf\logging.properties file as specified in the question above.
  8. The next time you start Tomcat in Eclipse, you should see the scanned .jars listed in the Eclipse Console instead of the "Enable debug logging for this logger" message. The information should also be logged in {TOMCAT_HOME}\logs\catalina.yyyy-mm-dd.log.

Media Player called in state 0, error (-38,0)

For me this worked

mp.seekTo(0);
mp.start();

Text-decoration: none not working

Try placing your text-decoration: none; on your a:hover css.

How to get CSS to select ID that begins with a string (not in Javascript)?

I'd do it like this:

[id^="product"] {
  ...
}

Ideally, use a class. This is what classes are for:

<div id="product176" class="product"></div>
<div id="product177" class="product"></div>
<div id="product178" class="product"></div>

And now the selector becomes:

.product {
  ...
}

How many times does each value appear in a column?

You can use CountIf. Put the following code in B1 and drag down the whole column

=COUNTIF(A:A,A1)

It will look like this:

enter image description here

What is the difference between _tmain() and main() in C++?

Ok, the question seems to have been answered fairly well, the UNICODE overload should take a wide character array as its second parameter. So if the command line parameter is "Hello" that would probably end up as "H\0e\0l\0l\0o\0\0\0" and your program would only print the 'H' before it sees what it thinks is a null terminator.

So now you may wonder why it even compiles and links.

Well it compiles because you are allowed to define an overload to a function.

Linking is a slightly more complex issue. In C, there is no decorated symbol information so it just finds a function called main. The argc and argv are probably always there as call-stack parameters just in case even if your function is defined with that signature, even if your function happens to ignore them.

Even though C++ does have decorated symbols, it almost certainly uses C-linkage for main, rather than a clever linker that looks for each one in turn. So it found your wmain and put the parameters onto the call-stack in case it is the int wmain(int, wchar_t*[]) version.

Change content of div - jQuery

Try this to Change content of div using jQuery.

See more @ Change content of div using jQuery

$(document).ready(function(){
    $("#Textarea").keyup(function(){
        // Getting the current value of textarea
        var currentText = $(this).val();

        // Setting the Div content
        $(".output").text(currentText);
    });
});

What is the point of "final class" in Java?

As above told, if you want no one can change the functionality of the method then you can declare it as final.

Example: Application server file path for download/upload, splitting string based on offset, such methods you can declare it Final so that these method functions will not be altered. And if you want such final methods in a separate class, then define that class as Final class. So Final class will have all final methods, where as Final method can be declared and defined in non-final class.

Connect over ssh using a .pem file

For AWS if the user is ubuntu use the following to connect to remote server.

chmod 400 mykey.pem

ssh -i mykey.pem ubuntu@your-ip

What do *args and **kwargs mean?

Notice the cool thing in S.Lott's comment - you can also call functions with *mylist and **mydict to unpack positional and keyword arguments:

def foo(a, b, c, d):
  print a, b, c, d

l = [0, 1]
d = {"d":3, "c":2}

foo(*l, **d)

Will print: 0 1 2 3

How to develop a soft keyboard for Android?

Create Custom Key Board for Own EditText

Download Entire Code

In this post i Created Simple Keyboard which contains Some special keys like ( France keys ) and it's supported Capital letters and small letters and Number keys and some Symbols .

package sra.keyboard;

import android.app.Activity;
import android.os.Bundle;
import android.util.Log;
import android.view.MotionEvent;
import android.view.View;
import android.view.WindowManager;
import android.view.View.OnClickListener;
import android.view.View.OnFocusChangeListener;
import android.view.View.OnTouchListener;
import android.widget.Button;
import android.widget.EditText;
import android.widget.RelativeLayout;

public class Main extends Activity implements OnTouchListener, OnClickListener,
  OnFocusChangeListener {
 private EditText mEt, mEt1; // Edit Text boxes
 private Button mBSpace, mBdone, mBack, mBChange, mNum;
 private RelativeLayout mLayout, mKLayout;
 private boolean isEdit = false, isEdit1 = false;
 private String mUpper = "upper", mLower = "lower";
 private int w, mWindowWidth;
 private String sL[] = { "a", "b", "c", "d", "e", "f", "g", "h", "i", "j",
   "k", "l", "m", "n", "o", "p", "q", "r", "s", "t", "u", "v", "w",
   "x", "y", "z", "ç", "à", "é", "è", "û", "î" };
 private String cL[] = { "A", "B", "C", "D", "E", "F", "G", "H", "I", "J",
   "K", "L", "M", "N", "O", "P", "Q", "R", "S", "T", "U", "V", "W",
   "X", "Y", "Z", "ç", "à", "é", "è", "û", "î" };
 private String nS[] = { "!", ")", "'", "#", "3", "$", "%", "&", "8", "*",
   "?", "/", "+", "-", "9", "0", "1", "4", "@", "5", "7", "(", "2",
   "\"", "6", "_", "=", "]", "[", "<", ">", "|" };
 private Button mB[] = new Button[32];

 @Override
 public void onCreate(Bundle savedInstanceState) {
  super.onCreate(savedInstanceState);
  try {
   setContentView(R.layout.main);
   // adjusting key regarding window sizes
   setKeys();
   setFrow();
   setSrow();
   setTrow();
   setForow();
   mEt = (EditText) findViewById(R.id.xEt);
   mEt.setOnTouchListener(this);
   mEt.setOnFocusChangeListener(this);
   mEt1 = (EditText) findViewById(R.id.et1);

   mEt1.setOnTouchListener(this);
   mEt1.setOnFocusChangeListener(this);
   mEt.setOnClickListener(this);
   mEt1.setOnClickListener(this);
   mLayout = (RelativeLayout) findViewById(R.id.xK1);
   mKLayout = (RelativeLayout) findViewById(R.id.xKeyBoard);

  } catch (Exception e) {
   Log.w(getClass().getName(), e.toString());
  }

 }

 @Override
 public boolean onTouch(View v, MotionEvent event) {
  if (v == mEt) {
   hideDefaultKeyboard();
   enableKeyboard();

  }
  if (v == mEt1) {
   hideDefaultKeyboard();
   enableKeyboard();

  }
  return true;
 }

 @Override
 public void onClick(View v) {

  if (v == mBChange) {

   if (mBChange.getTag().equals(mUpper)) {
    changeSmallLetters();
    changeSmallTags();
   } else if (mBChange.getTag().equals(mLower)) {
    changeCapitalLetters();
    changeCapitalTags();
   }

  } else if (v != mBdone && v != mBack && v != mBChange && v != mNum) {
   addText(v);

  } else if (v == mBdone) {

   disableKeyboard();

  } else if (v == mBack) {
   isBack(v);
  } else if (v == mNum) {
   String nTag = (String) mNum.getTag();
   if (nTag.equals("num")) {
    changeSyNuLetters();
    changeSyNuTags();
    mBChange.setVisibility(Button.INVISIBLE);

   }
   if (nTag.equals("ABC")) {
    changeCapitalLetters();
    changeCapitalTags();
   }

  }

 }

 @Override
 public void onFocusChange(View v, boolean hasFocus) {
  if (v == mEt && hasFocus == true) {
   isEdit = true;
   isEdit1 = false;

  } else if (v == mEt1 && hasFocus == true) {
   isEdit = false;
   isEdit1 = true;

  }

 }

 private void addText(View v) {
  if (isEdit == true) {
   String b = "";
   b = (String) v.getTag();
   if (b != null) {
    // adding text in Edittext
    mEt.append(b);

   }
  }

  if (isEdit1 == true) {
   String b = "";
   b = (String) v.getTag();
   if (b != null) {
    // adding text in Edittext
    mEt1.append(b);

   }

  }

 }

 private void isBack(View v) {
  if (isEdit == true) {
   CharSequence cc = mEt.getText();
   if (cc != null && cc.length() > 0) {
    {
     mEt.setText("");
     mEt.append(cc.subSequence(0, cc.length() - 1));
    }

   }
  }

  if (isEdit1 == true) {
   CharSequence cc = mEt1.getText();
   if (cc != null && cc.length() > 0) {
    {
     mEt1.setText("");
     mEt1.append(cc.subSequence(0, cc.length() - 1));
    }
   }
  }
 }
 private void changeSmallLetters() {
  mBChange.setVisibility(Button.VISIBLE);
  for (int i = 0; i < sL.length; i++)
   mB[i].setText(sL[i]);
  mNum.setTag("12#");
 }
 private void changeSmallTags() {
  for (int i = 0; i < sL.length; i++)
   mB[i].setTag(sL[i]);
  mBChange.setTag("lower");
  mNum.setTag("num");
 }
 private void changeCapitalLetters() {
  mBChange.setVisibility(Button.VISIBLE);
  for (int i = 0; i < cL.length; i++)
   mB[i].setText(cL[i]);
  mBChange.setTag("upper");
  mNum.setText("12#");

 }

 private void changeCapitalTags() {
  for (int i = 0; i < cL.length; i++)
   mB[i].setTag(cL[i]);
  mNum.setTag("num");

 }

 private void changeSyNuLetters() {

  for (int i = 0; i < nS.length; i++)
   mB[i].setText(nS[i]);
  mNum.setText("ABC");
 }

 private void changeSyNuTags() {
  for (int i = 0; i < nS.length; i++)
   mB[i].setTag(nS[i]);
  mNum.setTag("ABC");
 }

 // enabling customized keyboard
 private void enableKeyboard() {

  mLayout.setVisibility(RelativeLayout.VISIBLE);
  mKLayout.setVisibility(RelativeLayout.VISIBLE);

 }

 // Disable customized keyboard
 private void disableKeyboard() {
  mLayout.setVisibility(RelativeLayout.INVISIBLE);
  mKLayout.setVisibility(RelativeLayout.INVISIBLE);

 }

 private void hideDefaultKeyboard() {
  getWindow().setSoftInputMode(
    WindowManager.LayoutParams.SOFT_INPUT_STATE_ALWAYS_HIDDEN);

 }

 private void setFrow() {
  w = (mWindowWidth / 13);
  w = w - 15;
  mB[16].setWidth(w);
  mB[22].setWidth(w + 3);
  mB[4].setWidth(w);
  mB[17].setWidth(w);
  mB[19].setWidth(w);
  mB[24].setWidth(w);
  mB[20].setWidth(w);
  mB[8].setWidth(w);
  mB[14].setWidth(w);
  mB[15].setWidth(w);
  mB[16].setHeight(50);
  mB[22].setHeight(50);
  mB[4].setHeight(50);
  mB[17].setHeight(50);
  mB[19].setHeight(50);
  mB[24].setHeight(50);
  mB[20].setHeight(50);
  mB[8].setHeight(50);
  mB[14].setHeight(50);
  mB[15].setHeight(50);

 }

 private void setSrow() {
  w = (mWindowWidth / 10);
  mB[0].setWidth(w);
  mB[18].setWidth(w);
  mB[3].setWidth(w);
  mB[5].setWidth(w);
  mB[6].setWidth(w);
  mB[7].setWidth(w);
  mB[26].setWidth(w);
  mB[9].setWidth(w);
  mB[10].setWidth(w);
  mB[11].setWidth(w);
  mB[26].setWidth(w);

  mB[0].setHeight(50);
  mB[18].setHeight(50);
  mB[3].setHeight(50);
  mB[5].setHeight(50);
  mB[6].setHeight(50);
  mB[7].setHeight(50);
  mB[9].setHeight(50);
  mB[10].setHeight(50);
  mB[11].setHeight(50);
  mB[26].setHeight(50);
 }

 private void setTrow() {
  w = (mWindowWidth / 12);
  mB[25].setWidth(w);
  mB[23].setWidth(w);
  mB[2].setWidth(w);
  mB[21].setWidth(w);
  mB[1].setWidth(w);
  mB[13].setWidth(w);
  mB[12].setWidth(w);
  mB[27].setWidth(w);
  mB[28].setWidth(w);
  mBack.setWidth(w);

  mB[25].setHeight(50);
  mB[23].setHeight(50);
  mB[2].setHeight(50);
  mB[21].setHeight(50);
  mB[1].setHeight(50);
  mB[13].setHeight(50);
  mB[12].setHeight(50);
  mB[27].setHeight(50);
  mB[28].setHeight(50);
  mBack.setHeight(50);

 }

 private void setForow() {
  w = (mWindowWidth / 10);
  mBSpace.setWidth(w * 4);
  mBSpace.setHeight(50);
  mB[29].setWidth(w);
  mB[29].setHeight(50);

  mB[30].setWidth(w);
  mB[30].setHeight(50);

  mB[31].setHeight(50);
  mB[31].setWidth(w);
  mBdone.setWidth(w + (w / 1));
  mBdone.setHeight(50);

 }

 private void setKeys() {
  mWindowWidth = getWindowManager().getDefaultDisplay().getWidth(); // getting
  // window
  // height
  // getting ids from xml files
  mB[0] = (Button) findViewById(R.id.xA);
  mB[1] = (Button) findViewById(R.id.xB);
  mB[2] = (Button) findViewById(R.id.xC);
  mB[3] = (Button) findViewById(R.id.xD);
  mB[4] = (Button) findViewById(R.id.xE);
  mB[5] = (Button) findViewById(R.id.xF);
  mB[6] = (Button) findViewById(R.id.xG);
  mB[7] = (Button) findViewById(R.id.xH);
  mB[8] = (Button) findViewById(R.id.xI);
  mB[9] = (Button) findViewById(R.id.xJ);
  mB[10] = (Button) findViewById(R.id.xK);
  mB[11] = (Button) findViewById(R.id.xL);
  mB[12] = (Button) findViewById(R.id.xM);
  mB[13] = (Button) findViewById(R.id.xN);
  mB[14] = (Button) findViewById(R.id.xO);
  mB[15] = (Button) findViewById(R.id.xP);
  mB[16] = (Button) findViewById(R.id.xQ);
  mB[17] = (Button) findViewById(R.id.xR);
  mB[18] = (Button) findViewById(R.id.xS);
  mB[19] = (Button) findViewById(R.id.xT);
  mB[20] = (Button) findViewById(R.id.xU);
  mB[21] = (Button) findViewById(R.id.xV);
  mB[22] = (Button) findViewById(R.id.xW);
  mB[23] = (Button) findViewById(R.id.xX);
  mB[24] = (Button) findViewById(R.id.xY);
  mB[25] = (Button) findViewById(R.id.xZ);
  mB[26] = (Button) findViewById(R.id.xS1);
  mB[27] = (Button) findViewById(R.id.xS2);
  mB[28] = (Button) findViewById(R.id.xS3);
  mB[29] = (Button) findViewById(R.id.xS4);
  mB[30] = (Button) findViewById(R.id.xS5);
  mB[31] = (Button) findViewById(R.id.xS6);
  mBSpace = (Button) findViewById(R.id.xSpace);
  mBdone = (Button) findViewById(R.id.xDone);
  mBChange = (Button) findViewById(R.id.xChange);
  mBack = (Button) findViewById(R.id.xBack);
  mNum = (Button) findViewById(R.id.xNum);
  for (int i = 0; i < mB.length; i++)
   mB[i].setOnClickListener(this);
  mBSpace.setOnClickListener(this);
  mBdone.setOnClickListener(this);
  mBack.setOnClickListener(this);
  mBChange.setOnClickListener(this);
  mNum.setOnClickListener(this);

 }

}

Backup/Restore a dockerized PostgreSQL database

The below command can be used to take dump from docker postgress container

docker exec -t <postgres-container-name> pg_dump --no-owner -U <db-username> <db-name> > file-name-to-backup-to.sql

nginx: connect() failed (111: Connection refused) while connecting to upstream

I don't think that solution would work anyways because you will see some error message in your error log file.

The solution was a lot easier than what I thought.

simply, open the following path to your php5-fpm

sudo nano /etc/php5/fpm/pool.d/www.conf

or if you're the admin 'root'

nano /etc/php5/fpm/pool.d/www.conf

Then find this line and uncomment it:

listen.allowed_clients = 127.0.0.1

This solution will make you be able to use listen = 127.0.0.1:9000 in your vhost blocks

like this: fastcgi_pass 127.0.0.1:9000;

after you make the modifications, all you need is to restart or reload both Nginx and Php5-fpm

Php5-fpm

sudo service php5-fpm restart

or

sudo service php5-fpm reload

Nginx

sudo service nginx restart

or

sudo service nginx reload

From the comments:

Also comment

;listen = /var/run/php5-fpm.sock 

and add

listen = 9000

MS SQL 2008 - get all table names and their row counts in a DB

Posted for completeness.

If you are looking for row count of all tables in all databases (which was what I was looking for) then I found this combination of this and this to work. No idea whether it is optimal or not:

SET NOCOUNT ON
DECLARE @AllTables table (DbName sysname,SchemaName sysname, TableName sysname, RowsCount int )
DECLARE
     @SQL nvarchar(4000)
SET @SQL='SELECT ''?'' AS DbName, s.name AS SchemaName, t.name AS TableName, p.rows AS RowsCount FROM [?].sys.tables t INNER JOIN sys.schemas s ON t.schema_id=s.schema_id INNER JOIN [?].sys.partitions p ON p.OBJECT_ID = t.OBJECT_ID'

INSERT INTO @AllTables (DbName, SchemaName, TableName, RowsCount)
    EXEC sp_msforeachdb @SQL
SET NOCOUNT OFF
SELECT DbName, SchemaName, TableName, SUM(RowsCount), MIN(RowsCount), SUM(1)
FROM @AllTables
WHERE RowsCount > 0
GROUP BY DbName, SchemaName, TableName
ORDER BY DbName, SchemaName, TableName

Understanding events and event handlers in C#

publisher: where the events happen. Publisher should specify which delegate the class is using and generate necessary arguments, pass those arguments and itself to the delegate.

subscriber: where the response happen. Subscriber should specify methods to respond to events. These methods should take the same type of arguments as the delegate. Subscriber then add this method to publisher's delegate.

Therefore, when the event happen in publisher, delegate will receive some event arguments (data, etc), but publisher has no idea what will happen with all these data. Subscribers can create methods in their own class to respond to events in publisher's class, so that subscribers can respond to publisher's events.

How to make <label> and <input> appear on the same line on an HTML form?

For Bootstrap 4 it could be done with class="form-group" style="display: flex"

<div class="form-group" style="display: flex">
    <label>Topjava comment:</label>
    <input class="form-control" type="checkbox"/>
</div>

Count multiple columns with group by in one query

    SELECT SUM(Output.count),Output.attr 
FROM
(
    SELECT COUNT(column1  ) AS count,column1 AS attr FROM tab1 GROUP BY column1 
    UNION ALL
    SELECT COUNT(column2) AS count,column2 AS attr FROM tab1 GROUP BY column2
    UNION ALL
    SELECT COUNT(column3) AS count,column3 AS attr FROM tab1 GROUP BY column3) AS Output

    GROUP BY attr 

ASP.NET set hiddenfield a value in Javascript

document.getElementById('<%=hdntxtbxTaksit.ClientID%>').value

The id you set in server is the server id which is different from client id.

Cannot implicitly convert type from Task<>

You need to make TestGetMethod async too and attach await in front of GetIdList(); will unwrap the task to List<int>, So if your helper function is returning Task make sure you have await as you are calling the function async too.

public Task<List<int>> TestGetMethod()
{
    return GetIdList();
}    

async Task<List<int>> GetIdList()
{
    using (HttpClient proxy = new HttpClient())
    {
        string response = await proxy.GetStringAsync("www.test.com");
        List<int> idList = JsonConvert.DeserializeObject<List<int>>();
        return idList;
    }
}

Another option

public async void TestGetMethod(List<int> results)
{
    results = await GetIdList(); // await will unwrap the List<int>
}

How do I get an object's unqualified (short) class name?

A good old regex seems to be faster than the most of the previous shown methods:

// both of the below calls will output: ShortClassName

echo preg_replace('/.*\\\\/', '', 'ShortClassName');
echo preg_replace('/.*\\\\/', '', 'SomeNamespace\SomePath\ShortClassName');

So this works even when you provide a short class name or a fully qualified (canonical) class name.

What the regex does is that it consumes all previous chars until the last separator is found (which is also consumed). So the remaining string will be the short class name.

If you want to use a different separator (eg. / ) then just use that separator instead. Remember to escape the backslash (ie. \) and also the pattern char (ie. /) in the input pattern.

Get text from DataGridView selected cells

In this specific case, the ToString() will return the name of the object retruned by the SelectedCell Property.( a collection of the currently selected cells).

This behavior occurs when an object has no specific implenetation for the ToString() methods.

in our case, all you have to do is to iterate the collection of the cells and to accumulate its values to a string. then push this string to the TextBox.

have a look here how to implement the iteration:

msdn

How do you change the formatting options in Visual Studio Code?

I just found this extension called beautify in the Market Place and yes, it's another config\settings file. :)

Beautify javascript, JSON, CSS, Sass, and HTML in Visual Studio Code.

VS Code uses js-beautify internally, but it lacks the ability to modify the style you wish to use. This extension enables running js-beautify in VS Code, AND honouring any .jsbeautifyrc file in the open file's path tree to load your code styling. Run with F1 Beautify (to beautify a selection) or F1 Beautify file.

For help on the settings in the .jsbeautifyrc see Settings.md

Here is the GitHub repository: https://github.com/HookyQR/VSCodeBeautify

How to add data to DataGridView

LINQ is a "query" language (thats the Q), so modifying data is outside its scope.

That said, your DataGridView is presumably bound to an ItemsSource, perhaps of type ObservableCollection<T> or similar. In that case, just do something like X.ToList().ForEach(yourGridSource.Add) (this might have to be adapted based on the type of source in your grid).

data.frame Group By column

Using dplyr:

require(dplyr)    
df <- data.frame(A = c(1, 1, 2, 3, 3), B = c(2, 3, 3, 5, 6))
df %>% group_by(A) %>% summarise(B = sum(B))

## Source: local data frame [3 x 2]
## 
##   A  B
## 1 1  5
## 2 2  3
## 3 3 11

With sqldf:

library(sqldf)
sqldf('SELECT A, SUM(B) AS B FROM df GROUP BY A')

Similarity String Comparison in Java

You can also use z algorithm to find similarity in the string. Click here https://teakrunch.com/2020/05/09/string-similarity-hackerrank-challenge/

How can I restart a Java application?

System.err.println("Someone is Restarting me...");
setVisible(false);
try {
    Thread.sleep(600);
} catch (InterruptedException e1) {
    e1.printStackTrace();
}
setVisible(true);

I guess you don't really want to stop the application, but to "Restart" it. For that, you could use this and add your "Reset" before the sleep and after the invisible window.

Importing a long list of constants to a Python file

As an alternative to using the import approach described in several answers, have a look a the configparser module.

The ConfigParser class implements a basic configuration file parser language which provides a structure similar to what you would find on Microsoft Windows INI files. You can use this to write Python programs which can be customized by end users easily.

Basic authentication with fetch?

You can also use btoa instead of base64.encode().

headers.set('Authorization', 'Basic ' + btoa(username + ":" + password));

How to write a:hover in inline CSS?

This is the best code example:

_x000D_
_x000D_
<a_x000D_
 style="color:blue;text-decoration: underline;background: white;"_x000D_
 href="http://aashwin.com/index.php/education/library/"_x000D_
 onmouseover="this.style.color='#0F0'"_x000D_
 onmouseout="this.style.color='#00F'">_x000D_
   Library_x000D_
</a>
_x000D_
_x000D_
_x000D_

Moderator Suggestion: Keep your separation of concerns.

HTML

_x000D_
_x000D_
<a_x000D_
 style="color:blue;text-decoration: underline;background: white;"_x000D_
 href="http://aashwin.com/index.php/education/library/"_x000D_
 class="lib-link">_x000D_
   Library_x000D_
</a>
_x000D_
_x000D_
_x000D_

JS

_x000D_
_x000D_
const libLink = document.getElementsByClassName("lib-link")[0];_x000D_
// The array 0 assumes there is only one of these links,_x000D_
// you would have to loop or use event delegation for multiples_x000D_
// but we won't go into that here_x000D_
libLink.onmouseover = function () {_x000D_
  this.style.color='#0F0'_x000D_
}_x000D_
libLink.onmouseout = function () {_x000D_
  this.style.color='#00F'_x000D_
}
_x000D_
_x000D_
_x000D_

SQL Select between dates

Change your data to that formats to use sqlite datetime formats.

YYYY-MM-DD
YYYY-MM-DD HH:MM
YYYY-MM-DD HH:MM:SS
YYYY-MM-DD HH:MM:SS.SSS
YYYY-MM-DDTHH:MM
YYYY-MM-DDTHH:MM:SS
YYYY-MM-DDTHH:MM:SS.SSS
HH:MM
HH:MM:SS
HH:MM:SS.SSS
now
DDDDDDDDDD

SELECT * FROM test WHERE date BETWEEN '2011-01-11' AND '2011-08-11'

How to get the full url in Express?

Use this,

var url = req.headers.host + '/' + req.url;

Git is not working after macOS Update (xcrun: error: invalid active developer path (/Library/Developer/CommandLineTools)

updated from Mojave to Big Sur and got the same error : the command

xcode-select --install

worked like a charm

"pip install json" fails on Ubuntu

json is a built-in module, you don't need to install it with pip.

Write a number with two decimal places SQL Server

try this

SELECT CONVERT(DECIMAL(10,2),YOURCOLUMN)

Extract data from XML Clob using SQL from Oracle Database

Try

SELECT EXTRACTVALUE(xmltype(testclob), '/DCResponse/ContextData/Field[@key="Decision"]') 
FROM traptabclob;

Here is a sqlfiddle demo

How to make <input type="date"> supported on all browsers? Any alternatives?

best easy and working solution i have found is, working on following browsers

  1. Google Chrome
  2. Firefox
  3. Microsoft Edge
  4. Safari

    <!doctype html>
    <html>
    <head>
    <meta charset="utf-8">
    <title>Untitled Document</title>
    </head>
    
    <body>
    <h2>Poly Filler Script for Date/Time</h2>
    <form method="post" action="">
        <input type="date" />
        <br/><br/>
        <input type="time" />
    </form>
    
    <script src="https://ajax.googleapis.com/ajax/libs/jquery/1.11.3/jquery.min.js"></script>
    <script src="http://cdn.jsdelivr.net/webshim/1.12.4/extras/modernizr-custom.js"></script>
    <script src="http://cdn.jsdelivr.net/webshim/1.12.4/polyfiller.js"></script>
    <script>
      webshims.setOptions('waitReady', false);
      webshims.setOptions('forms-ext', {type: 'date'});
      webshims.setOptions('forms-ext', {type: 'time'});
      webshims.polyfill('forms forms-ext');
    </script>
    
    </body>
    </html>
    

How to make the background image to fit into the whole page without repeating using plain css?

background:url(bgimage.jpg) no-repeat; background-size: cover;

This did the trick

How to pass json POST data to Web API method as an object?

Make sure that your WebAPI service is expecting a strongly typed object with a structure that matches the JSON that you are passing. And make sure that you stringify the JSON that you are POSTing.

Here is my JavaScript (using AngluarJS):

$scope.updateUserActivity = function (_objuserActivity) {
        $http
        ({
            method: 'post',
            url: 'your url here',
            headers: { 'Content-Type': 'application/json'},
            data: JSON.stringify(_objuserActivity)
        })
        .then(function (response)
        {
            alert("success");
        })
        .catch(function (response)
        {
            alert("failure");
        })
        .finally(function ()
        {
        });

And here is my WebAPI Controller:

[HttpPost]
[AcceptVerbs("POST")]
public string POSTMe([FromBody]Models.UserActivity _activity)
{
    return "hello";
}

jQuery + client-side template = "Syntax error, unrecognized expression"

As the official document: As of 1.9, a string is only considered to be HTML if it starts with a less-than ("<") character. The Migrate plugin can be used to restore the pre-1.9 behavior.

If a string is known to be HTML but may start with arbitrary text that is not an HTML tag, pass it to jQuery.parseHTML() which will return an array of DOM nodes representing the markup. A jQuery collection can be created from this, for example: $($.parseHTML(htmlString)). This would be considered best practice when processing HTML templates for example. Simple uses of literal strings such as $("<p>Testing</p>").appendTo("body") are unaffected by this change.

How to disable sort in DataGridView?

foreach (DataGridViewColumn column in dataGridView.Columns)
{
    column.SortMode = DataGridViewColumnSortMode.NotSortable;
}

iterate through a map in javascript

Functional Approach for ES6+

If you want to take a more functional approach to iterating over the Map object, you can do something like this

const myMap = new Map() 
myMap.forEach((value, key) => {
    console.log(value, key)
})

SQL RANK() over PARTITION on joined tables

As the rank doesn't depend at all from the contacts

RANKED_RSLTS

 QRY_ID  |  RES_ID  |  SCORE |  RANK
-------------------------------------
   A     |    1     |    15  |   3
   A     |    2     |    32  |   1
   A     |    3     |    29  |   2
   C     |    7     |    61  |   1
   C     |    9     |    30  |   2

Thus :

SELECT
    C.*
    ,R.SCORE
    ,MYRANK
FROM CONTACTS C LEFT JOIN
(SELECT  *,
 MYRANK = RANK() OVER (PARTITION BY QRY_ID ORDER BY SCORE DESC)
  FROM RSLTS)  R
ON C.RES_ID = R.RES_ID
AND C.QRY_ID = R.QRY_ID

How to convert an array to object in PHP?

I also had this issue, but I noticed that json_decode converts JSON array to object.

So, I came about my solution by using json_encode($PHPArray) which returns A JSON string of object, then I decoded the string with Json_decode($string) and it would return a perfectly structured object. Shorthand

$object = json_decode(json_encode($array));

Or

$jsonString = json_encode($array);
$object = json_decode($jsonString);

Apache: "AuthType not set!" 500 Error

Remove the line that says

Require all granted

it's only needed on Apache >=2.4

Python UTC datetime object's ISO format doesn't include Z (Zulu or Zero offset)

By combining all answers above I came with following function :

from datetime import datetime, tzinfo, timedelta
class simple_utc(tzinfo):
    def tzname(self,**kwargs):
        return "UTC"
    def utcoffset(self, dt):
        return timedelta(0)


def getdata(yy, mm, dd, h, m, s) :
    d = datetime(yy, mm, dd, h, m, s)
    d = d.replace(tzinfo=simple_utc()).isoformat()
    d = str(d).replace('+00:00', 'Z')
    return d


print getdata(2018, 02, 03, 15, 0, 14)

jquery remove "selected" attribute of option?

Using jQuery 1.9 and above:

$("#mySelect :selected").prop('selected', false);

Export specific rows from a PostgreSQL table as INSERT SQL script

This is an easy and fast way to export a table to a script with pgAdmin manually without extra installations:

  1. Right click on target table and select "Backup".
  2. Select a file path to store the backup. As Format choose "Plain".
  3. Open the tab "Dump Options #2" at the bottom and check "Use Column Inserts".
  4. Click the Backup-button.
  5. If you open the resulting file with a text reader (e.g. notepad++) you get a script to create the whole table. From there you can simply copy the generated INSERT-Statements.

This method also works with the technique of making an export_table as demonstrated in @Clodoaldo Neto's answer.

Click right on target table and choose "Backup"

Choose a destination path and change the format to "Plain"

Open the tab "Dump Options #2" at the bottom and check "Use Column Inserts"

You can copy the INSERT Statements from there.

Not able to access adb in OS X through Terminal, "command not found"

  1. run command in terminal nano $HOME/.zshrc

  2. Must include next lines:

    export PATH=$PATH:~/Library/Android/sdk/platform-tools
    export ANDROID_HOME=~/Library/Android/sdk
    export PATH="$HOME/.bin:$PATH"
    export PATH="~/Library/Android/sdk/platform-tools":$PATH
    
  3. Press Command + X to save file in editor,Enter Yes or No and hit Enter key

  4. Run source ~/.zshrc

  5. Check adb in terminal, run adb

C# naming convention for constants?

First, Hungarian Notation is the practice of using a prefix to display a parameter's data type or intended use. Microsoft's naming conventions for says no to Hungarian Notation http://en.wikipedia.org/wiki/Hungarian_notation http://msdn.microsoft.com/en-us/library/ms229045.aspx

Using UPPERCASE is not encouraged as stated here: Pascal Case is the acceptable convention and SCREAMING CAPS. http://en.wikibooks.org/wiki/C_Sharp_Programming/Naming

Microsoft also states here that UPPERCASE can be used if it is done to match the the existed scheme. http://msdn.microsoft.com/en-us/library/x2dbyw72.aspx

This pretty much sums it up.

filters on ng-model in an input

I would suggest to watch model value and update it upon chage: http://plnkr.co/edit/Mb0uRyIIv1eK8nTg3Qng?p=preview

The only interesting issue is with spaces: In AngularJS 1.0.3 ng-model on input automatically trims string, so it does not detect that model was changed if you add spaces at the end or at start (so spaces are not automatically removed by my code). But in 1.1.1 there is 'ng-trim' directive that allows to disable this functionality (commit). So I've decided to use 1.1.1 to achieve exact functionality you described in your question.

How can I fix the form size in a C# Windows Forms application and not to let user change its size?

I'm pretty sure this isn't the BEST way, but you could set the MinimumSize and MaximimSize properties to the same value. That will stop it.

get dataframe row count based on conditions

In Pandas, I like to use the shape attribute to get number of rows.

df[df.A > 0].shape[0]

gives the number of rows matching the condition A > 0, as desired.

Convert JSON String To C# Object

Using dynamic object with JavaScriptSerializer.

JavaScriptSerializer serializer = new JavaScriptSerializer(); 
dynamic item = serializer.Deserialize<object>("{ \"test\":\"some data\" }");
string test= item["test"];

//test Result = "some data"

Mac install and open mysql using terminal

In the terminal, I typed:

/usr/local/mysql/bin/mysql -u root -p

I was then prompted to enter the temporary password that was given to me upon completion of the installation.

find a minimum value in an array of floats

If you want to use numpy, you must define darr to be a numpy array, not a list:

import numpy as np
darr = np.array([1, 3.14159, 1e100, -2.71828])
print(darr.min())

darr.argmin() will give you the index corresponding to the minimum.

The reason you were getting an error is because argmin is a method understood by numpy arrays, but not by Python lists.

git rm - fatal: pathspec did not match any files

To remove the tracked and old committed file from git you can use the below command. Here in my case, I want to untrack and remove all the file from dist directory.

git filter-branch --force --index-filter 'git rm -r --cached --ignore-unmatch dist' --tag-name-filter cat -- --all

Then, you need to add it into your .gitignore so it won't be tracked further.

Sending commands and strings to Terminal.app with Applescript

Your question is specifically about how to get Applescript to do what you want. But, for the particular example described, you might want to look into 'expect' as a solution.

Reading data from XML

as per @Jon Skeet 's comment, you should use a XmlReader only if your file is very big. Here's how to use it. Assuming you have a Book class

public class Book {
    public string Title {get; set;}
    public string Author {get; set;}
}

you can read the XML file line by line with a small memory footprint, like this:

public static class XmlHelper {
    public static IEnumerable<Book> StreamBooks(string uri) {
        using (XmlReader reader = XmlReader.Create(uri)) {
            string title = null;
            string author = null;

            reader.MoveToContent();
            while (reader.Read()) {
                if (reader.NodeType == XmlNodeType.Element
                    && reader.Name == "Book") {
                    while (reader.Read()) {
                        if (reader.NodeType == XmlNodeType.Element &&
                            reader.Name == "Title") {
                            title = reader.ReadString();
                            break;
                        }
                    }
                    while (reader.Read()) {
                        if (reader.NodeType == XmlNodeType.Element &&
                            reader.Name == "Author") {
                            author =reader.ReadString();
                            break;
                        }
                    }
                    yield return new Book() {Title = title, Author = author};
                }
            }       
        }
    }

Example of usage:

string uri = @"c:\test.xml"; // your big XML file

foreach (var book in XmlHelper.StreamBooks(uri)) {
    Console.WriteLine("Title, Author: {0}, {1}", book.Title, book.Author);  
}

#include errors detected in vscode

I ended up here after struggling for a while, but actually what I was missing was just:

If a #include file or one of its dependencies cannot be found, you can also click on the red squiggles under the include statements to view suggestions for how to update your configuration.

enter image description here

source: https://code.visualstudio.com/docs/languages/cpp#_intellisense

jQuery show for 5 seconds then hide

You can use .delay() before an animation, like this:

$("#myElem").show().delay(5000).fadeOut();

If it's not an animation, use setTimeout() directly, like this:

$("#myElem").show();
setTimeout(function() { $("#myElem").hide(); }, 5000);

You do the second because .hide() wouldn't normally be on the animation (fx) queue without a duration, it's just an instant effect.

Or, another option is to use .delay() and .queue() yourself, like this:

$("#myElem").show().delay(5000).queue(function(n) {
  $(this).hide(); n();
});

Using node.js as a simple web server

There are already some great solutions for a simple nodejs server. There is a one more solution if you need live-reloading as you made changes to your files.

npm install lite-server -g

navigate your directory and do

lite-server

it will open browser for you with live-reloading.

How to run VBScript from command line without Cscript/Wscript

Why don't you just stash the vbscript in a batch/vbscript file hybrid. Name the batch hybrid Converter.bat and you can execute it directly as Converter from the cmd line. Sure you can default ALL scripts to run from Cscript or Wscript, but if you want to execute your vbs as a windows script rather than a console script, this could cause some confusion later on. So just set your code to a batch file and run it directly.

Check the answer -> Here

And here is an example:

Converter.bat

::' VBS/Batch Hybrid
::' --- Batch portion ---------
rem^ &@echo off
rem^ &call :'sub
rem^ &exit /b

:'sub
rem^ &echo begin batch
rem^ &cscript //nologo //e:vbscript "%~f0"
rem^ &echo end batch
rem^ &exit /b

'----- VBS portion -----
Dim tester
tester = "Convert data here"
Msgbox tester

Change one value based on another value in pandas

One option is to use Python's slicing and indexing features to logically evaluate the places where your condition holds and overwrite the data there.

Assuming you can load your data directly into pandas with pandas.read_csv then the following code might be helpful for you.

import pandas
df = pandas.read_csv("test.csv")
df.loc[df.ID == 103, 'FirstName'] = "Matt"
df.loc[df.ID == 103, 'LastName'] = "Jones"

As mentioned in the comments, you can also do the assignment to both columns in one shot:

df.loc[df.ID == 103, ['FirstName', 'LastName']] = 'Matt', 'Jones'

Note that you'll need pandas version 0.11 or newer to make use of loc for overwrite assignment operations.


Another way to do it is to use what is called chained assignment. The behavior of this is less stable and so it is not considered the best solution (it is explicitly discouraged in the docs), but it is useful to know about:

import pandas
df = pandas.read_csv("test.csv")
df['FirstName'][df.ID == 103] = "Matt"
df['LastName'][df.ID == 103] = "Jones"

For vs. while in C programming?

One peculiarity of the do while is that you need a semi-colon after the while to complete. It is often used in macro definitions to execute several statements only once while constraining the impact of the macro. If macros where defined as blocks, some parsing errors may occur.

One explanation among others

How do you change Background for a Button MouseOver in WPF?

Just want to share my button style from my ResourceDictionary that i've been using. You can freely change the onHover background at the style triggers. "ColorAnimation To = *your desired BG(i.e #FFCEF7A0)". The button BG will also automatically revert to its original BG after the mouseOver state.You can even set how fast the transition.

Resource Dictionary

<Style x:Key="Flat_Button" TargetType="{x:Type Button}">
    <Setter Property="Width" Value="100"/>
    <Setter Property="Height" Value="50"/>
    <Setter Property="Margin" Value="2"/>
    <Setter Property="FontFamily" Value="Arial Narrow"/>
    <Setter Property="FontSize" Value="12px"/>
    <Setter Property="FontWeight" Value="Bold"/>
    <Setter Property="Cursor" Value="Hand"/>
    <Setter Property="Foreground">
        <Setter.Value>
            <SolidColorBrush Opacity="1" Color="White"/>
        </Setter.Value>
    </Setter>
    <Setter Property="Background" >
        <Setter.Value>
            <SolidColorBrush Opacity="1" Color="#28C2FF" />
        </Setter.Value>
    </Setter>
    <Setter Property="Template">
        <Setter.Value>
            <ControlTemplate TargetType="{x:Type Button}">

                <Border x:Name="border"
                         SnapsToDevicePixels="True"
                         BorderThickness="1"
                         Padding="4,2"
                         BorderBrush="Gray"
                         CornerRadius="3"
                         Background="{TemplateBinding Background}">
                    <Grid>
                        <ContentPresenter 
                        Margin="2"
                        HorizontalAlignment="Center"
                        VerticalAlignment="Center"
                        RecognizesAccessKey="True" />

                    </Grid>
                </Border>

            </ControlTemplate>
        </Setter.Value>
    </Setter>

    <Style.Triggers>
        <Trigger Property="IsMouseOver" Value="true">
            <Trigger.EnterActions>
                <BeginStoryboard>
                    <Storyboard>
                        <ColorAnimation To="#D2F898"
                                        Storyboard.TargetProperty="(Control.Background).(SolidColorBrush.Color)" 
                                        FillBehavior="HoldEnd" Duration="0:0:0.25" AutoReverse="False" RepeatBehavior="1x"/>
                    </Storyboard>
                </BeginStoryboard>
            </Trigger.EnterActions>

            <Trigger.ExitActions>
                <BeginStoryboard>
                    <Storyboard>
                        <ColorAnimation
                                            Storyboard.TargetProperty="(Control.Background).(SolidColorBrush.Color)" 
                                            FillBehavior="HoldEnd" Duration="0:0:0.25" AutoReverse="False" RepeatBehavior="1x"/>
                    </Storyboard>
                </BeginStoryboard>
            </Trigger.ExitActions>

        </Trigger>


    </Style.Triggers>
</Style>

all you have to do is call the style.

Example Implementation

<Button Style="{StaticResource Flat_Button}" Height="Auto"Width="Auto">  
     <StackPanel>
     <TextBlock Text="SAVE" FontFamily="Arial" FontSize="10.667"/>
     </StackPanel>
</Button>

How do I dynamically change the content in an iframe using jquery?

<html>
  <head>
    <script type="text/javascript" src="jquery.js"></script>
    <script>
      $(document).ready(function(){
        var locations = ["http://webPage1.com", "http://webPage2.com"];
        var len = locations.length;
        var iframe = $('#frame');
        var i = 0;
        setInterval(function () {
            iframe.attr('src', locations[++i % len]);
        }, 30000);
      });
    </script>
  </head>
  <body>
    <iframe id="frame"></iframe>
  </body>
</html>

Entity Framework (EF) Code First Cascade Delete for One-to-Zero-or-One relationship

This code worked for me

protected override void OnModelCreating(DbModelBuilder modelBuilder)
    {
        modelBuilder.Entity<UserDetail>()
            .HasRequired(d => d.User)
            .WithOptional(u => u.UserDetail)
            .WillCascadeOnDelete(true);
    }

The migration code was:

public override void Up()
    {
        AddForeignKey("UserDetail", "UserId", "User", "UserId", cascadeDelete: true);
    }

And it worked fine. When I first used

modelBuilder.Entity<User>()
    .HasOptional(a => a.UserDetail)
    .WithOptionalDependent()
    .WillCascadeOnDelete(true);

The migration code was:

AddForeignKey("User", "UserDetail_UserId", "UserDetail", "UserId", cascadeDelete: true); 

but it does not match any of the two overloads available (in EntityFramework 6)

Running Composer returns: "Could not open input file: composer.phar"

Hi friends, follow the steps to fix this issue in MAC OS

  • Step 1: first run the command in Terminal with your project directory

    $ curl -sS https://getcomposer.org/installer | php

  • Step 2: Move the composer.phar in your project directory

    $ mv composer.phar /Applications/MAMP/htdocs/bashrushAPI/composer.phar

  • Step 3: Setup alias the composer

    $ alias composer='/Applications/MAMP/htdocs/bashrushAPI/composer.phar'

  • Step 4: Check the composer version now

    $ composer --version

    Composer version 1.7.2 2018-08-16 16:57:12

  • Step 5: Confirm the project folders and file placed on bellow

    $ ls

    CONTRIBUTING.md docker-compose.yml templates README.md logs tests composer.json phpunit.xml vendor composer.lock public composer.phar src

  • Step 6: Now update composer

    $ composer.phar update

    Loading composer repositories with package information Updating dependencies (including require-dev) Nothing to install or update Generating autoload files

  • Step 7: Run your sample project

    $ php composer.phar start

    php -S localhost:8080 -t public [Thu Sep 27 03:16:11 2018] ::1:51177 [200]: / [Thu Sep 27 03:16:11 2018] ::1:51178 [404]: /favicon.ico - No such file or directory

while installing vc_redist.x64.exe, getting error "Failed to configure per-machine MSU package."

I faced a similar problem but in my case I was trying to install Visual C++ Redistributable for Visual Studio 2015 Update 1 on Windows Server 2012 R2. However the root cause should be the same.

In short, you need to install the prerequisites of KB2999226.

In more details, the installation log I got stated that the installation for Windows Update KB2999226 failed. According to the Microsoft website here:

Prerequisites To install this update, you must have April 2014 update rollup for Windows RT 8.1, Windows 8.1, and Windows Server 2012 R2 (2919355) installed in Windows 8.1 or Windows Server 2012 R2. Or, install Service Pack 1 for Windows 7 or Windows Server 2008 R2. Or, install Service Pack 2 for Windows Vista and for Windows Server 2008.

After I have installed April 2014 on my Windows Server 2012 R2, I am able to install the Visual C++ Redistributable correctly.

Why do I get "Pickle - EOFError: Ran out of input" reading an empty file?

As you see, that's actually a natural error ..

A typical construct for reading from an Unpickler object would be like this ..

try:
    data = unpickler.load()
except EOFError:
    data = list()  # or whatever you want

EOFError is simply raised, because it was reading an empty file, it just meant End of File ..

Is there any kind of hash code function in JavaScript?

My solution introduces a static function for the global Object object.

(function() {
    var lastStorageId = 0;

    this.Object.hash = function(object) {
        var hash = object.__id;

        if (!hash)
             hash = object.__id = lastStorageId++;

        return '#' + hash;
    };
}());

I think this is more convenient with other object manipulating functions in JavaScript.

How to run a program automatically as admin on Windows 7 at startup?

You should also consider the security implications of running a process as an administrator level user or as Service. If any input is not being validated properly, such as if it is listening on a network interface. If the parser for this input doesn't validate properly, it can be abused, and possibly lead to an exploit that could run code as the elevated user. in abatishchev's example it shouldn't be much of a problem, but if it were to be deployed in an enterprise environment, do a security assessment prior to wide scale deployment.

How to determine if a point is in a 2D triangle?

Here is a solution in python that is efficient, documented and contains three unittests. It's professional-grade quality and ready to be dropped into your project in the form of a module as is.

import unittest

###############################################################################
def point_in_triangle(point, triangle):
    """Returns True if the point is inside the triangle
    and returns False if it falls outside.
    - The argument *point* is a tuple with two elements
    containing the X,Y coordinates respectively.
    - The argument *triangle* is a tuple with three elements each
    element consisting of a tuple of X,Y coordinates.

    It works like this:
    Walk clockwise or counterclockwise around the triangle
    and project the point onto the segment we are crossing
    by using the dot product.
    Finally, check that the vector created is on the same side
    for each of the triangle's segments.
    """
    # Unpack arguments
    x, y = point
    ax, ay = triangle[0]
    bx, by = triangle[1]
    cx, cy = triangle[2]
    # Segment A to B
    side_1 = (x - bx) * (ay - by) - (ax - bx) * (y - by)
    # Segment B to C
    side_2 = (x - cx) * (by - cy) - (bx - cx) * (y - cy)
    # Segment C to A
    side_3 = (x - ax) * (cy - ay) - (cx - ax) * (y - ay)
    # All the signs must be positive or all negative
    return (side_1 < 0.0) == (side_2 < 0.0) == (side_3 < 0.0)

###############################################################################
class TestPointInTriangle(unittest.TestCase):

    triangle = ((22 , 8),
                (12 , 55),
                (7 , 19))

    def test_inside(self):
        point = (15, 20)
        self.assertTrue(point_in_triangle(point, self.triangle))

    def test_outside(self):
        point = (1, 7)
        self.assertFalse(point_in_triangle(point, self.triangle))

    def test_border_case(self):
        """If the point is exactly on one of the triangle's edges,
        we consider it is inside."""
        point = (7, 19)
        self.assertTrue(point_in_triangle(point, self.triangle))

###############################################################################
if __name__ == "__main__":
    suite = unittest.defaultTestLoader.loadTestsFromTestCase(TestPointInTriangle)
    unittest.TextTestRunner().run(suite)

There is an additional optional graphical test for the algorithm above to confirm its validity:

import random
from matplotlib import pyplot
from triangle_test import point_in_triangle

###############################################################################
# The area #
size_x = 64
size_y = 64

# The triangle #
triangle = ((22 , 8),
            (12 , 55),
            (7 , 19))

# Number of random points #
count_points = 10000

# Prepare the figure #
figure = pyplot.figure()
axes = figure.add_subplot(111, aspect='equal')
axes.set_title("Test the 'point_in_triangle' function")
axes.set_xlim(0, size_x)
axes.set_ylim(0, size_y)

# Plot the triangle #
from matplotlib.patches import Polygon
axes.add_patch(Polygon(triangle, linewidth=1, edgecolor='k', facecolor='none'))

# Plot the points #
for i in range(count_points):
    x = random.uniform(0, size_x)
    y = random.uniform(0, size_y)
    if point_in_triangle((x,y), triangle): pyplot.plot(x, y, '.g')
    else:                                  pyplot.plot(x, y, '.b')

# Save it #
figure.savefig("point_in_triangle.pdf")

Producing the following graphic:

Test the point_in_triangle function

How to detect scroll direction

var mousewheelevt = (/Firefox/i.test(navigator.userAgent)) ? "DOMMouseScroll" : "mousewheel" //FF doesn't recognize mousewheel as of FF3.x
$(document).bind(mousewheelevt, 
function(e)
    {
        var evt = window.event || e //equalize event object     
        evt = evt.originalEvent ? evt.originalEvent : evt; //convert to originalEvent if possible               
        var delta = evt.detail ? evt.detail*(-40) : evt.wheelDelta //check for detail first, because it is used by Opera and FF
        if(delta > 0) 
            {
            scrollup();
            }
        else
            {
            scrolldown();
            }   
    }
);

how to get data from selected row from datagridview

To get the cell value, you need to read it directly from DataGridView1 using e.RowIndex and e.ColumnIndex properties.

Eg:

Private Sub DataGridView1_CellContentClick(ByVal sender As System.Object, ByVal e As System.Windows.Forms.DataGridViewCellEventArgs) Handles DataGridView1.CellContentClick
   Dim value As Object = DataGridView1.Rows(e.RowIndex).Cells(e.ColumnIndex).Value

   If IsDBNull(value) Then 
      TextBox1.Text = "" ' blank if dbnull values
   Else
      TextBox1.Text = CType(value, String)
   End If
End Sub

How to create a label inside an <input> element?

When you start typing it will disappear.If empty it will appear again.

        <%= f.text_field :user_email,:value=>"",:placeholder => "Eg:[email protected]"%>

Simplest way...

How to share my Docker-Image without using the Docker-Hub?

Docker images are stored as filesystem layers. Every command in the Dockerfile creates a layer. You can also create layers by using docker commit from the command line after making some changes (via docker run probably).

These layers are stored by default under /var/lib/docker. While you could (theoretically) cherry pick files from there and install it in a different docker server, is probably a bad idea to play with the internal representation used by Docker.

When you push your image, these layers are sent to the registry (the docker hub registry, by default… unless you tag your image with another registry prefix) and stored there. When pushing, the layer id is used to check if you already have the layer locally or it needs to be downloaded. You can use docker history to peek at which layers (other images) are used (and, to some extent, which command created the layer).

As for options to share an image without pushing to the docker hub registry, your best options are:

  • docker save an image or docker export a container. This will output a tar file to standard output, so you will like to do something like docker save 'dockerizeit/agent' > dk.agent.latest.tar. Then you can use docker load or docker import in a different host.

  • Host your own private registry. - Outdated, see comments See the docker registry image. We have built an s3 backed registry which you can start and stop as needed (all state is kept on the s3 bucket of your choice) which is trivial to setup. This is also an interesting way of watching what happens when pushing to a registry

  • Use another registry like quay.io (I haven't personally tried it), although whatever concerns you have with the docker hub will probably apply here too.

How to set Toolbar text and back arrow color

<android.support.v7.widget.Toolbar

xmlns:android="http://schemas.android.com/apk/res/android"


xmlns:app="http://schemas.android.com/apk/res-auto"


android:id="@+id/toolbar"


android:layout_width="match_parent"


android:layout_height="wrap_content"


android:background="@color/color_primary"


android:theme="@style/GalaxyZooThemeToolbarDarkOverflow"


app:popupTheme="@style/Theme.AppCompat.NoActionBar" />

That toolbar theme specifies a textColorPrimary and textColorSecondary to change the color of the title text and of the menu overflow button. You could just specify the standard Theme.AppCompat.Light.NoActionBar theme for the toolbar, to get the dark text and overflow icon, but I wanted to derive from my own theme and make only small changes, because I have no idea what else might be affected.

<style name="GalaxyZooThemeToolbarDarkOverflow" parent="Theme.AppCompat.NoActionBar">


<!-- android:textColorPrimary is the  color of the title text
   in the Toolbar, in the Theme.AppCompat theme:  -->


<item name="android:textColorPrimary">@color/abc_primary_text_material_light</item>



<!-- android:textColorPrimaryInverse is the  color of the title
   text in the Toolbar, in the Theme.AppCompat.Light theme:  -->


<!-- <item name="android:textColorPrimaryInverse">@color/abc_primary_text_material_light</item> -->

This is how you can change the color of text and icons on toolbar.

For more Ref: https://www.murrayc.com/permalink/2014/10/28/android-changing-the-toolbars-text-color-and-overflow-icon-color/

The target ... overrides the `OTHER_LDFLAGS` build setting defined in `Pods/Pods.xcconfig

The first line of link below saved my day:

To add values to options from your project’s build settings, prepend the value list with $(inherited).

https://github.com/CocoaPods/CocoaPods/wiki/Creating-a-project-that-uses-CocoaPods#faq

Also, do not forget to insert this line at the beginning of your pod file:

platform :iOS, '5.0'

Compiler error: "class, interface, or enum expected"

class, interface, or enum expected

The above error is even possible when import statement is miss spelled. A proper statement is "import com.company.HelloWorld;"

If by mistake while code writing/editing it is miss written like "t com.company.HelloWorld;"

compiler will show "class, interface, or enum expected"

How can I pass a member function where a free function is expected?

You can stop banging your heads now. Here is the wrapper for the member function to support existing functions taking in plain C functions as arguments. thread_local directive is the key here.

http://cpp.sh/9jhk3

// Example program
#include <iostream>
#include <string>

using namespace std;

typedef int FooCooker_ (int);

// Existing function
extern "C" void cook_10_foo (FooCooker_ FooCooker) {
    cout << "Cooking 10 Foo ..." << endl;
    cout << "FooCooker:" << endl;
    FooCooker (10);
}

struct Bar_ {
    Bar_ (int Foo = 0) : Foo (Foo) {};
    int cook (int Foo) {
        cout << "This Bar got " << this->Foo << endl;
        if (this->Foo >= Foo) {
            this->Foo -= Foo;
            cout << Foo << " cooked" << endl;
            return Foo;
        } else {
            cout << "Can't cook " <<  Foo << endl;
            return 0;
        }
    }
    int Foo = 0;
};

// Each Bar_ object and a member function need to define
// their own wrapper with a global thread_local object ptr
// to be called as a plain C function.
thread_local static Bar_* BarPtr = NULL;
static int cook_in_Bar (int Foo) {
    return BarPtr->cook (Foo);
}

thread_local static Bar_* Bar2Ptr = NULL;
static int cook_in_Bar2 (int Foo) {
    return Bar2Ptr->cook (Foo);
}

int main () {
  BarPtr = new Bar_ (20);
  cook_10_foo (cook_in_Bar);

  Bar2Ptr = new Bar_ (40);
  cook_10_foo (cook_in_Bar2);

  delete BarPtr;
  delete Bar2Ptr;
  return 0;
}

Please comment on any issues with this approach.

Other answers fail to call existing plain C functions: http://cpp.sh/8exun

How to hide a <option> in a <select> menu with CSS?

!!! WARNING !!!

Replace the second "IF" by "WHILE" or doesn't work !

jQuery.fn.toggleOption = function( show ) {
    jQuery( this ).toggle( show );
    if( show ) {
        while( jQuery( this ).parent( 'span.toggleOption' ).length )
            jQuery( this ).unwrap( );
    } else {
        jQuery( this ).wrap( '<span class="toggleOption" style="display: none;" />' );
    }
};

How to modify WooCommerce cart, checkout pages (main theme portion)

I've found this works well as a conditional within page.php that includes the WooCommerce cart and checkout screens.

!is_page(array('cart', 'checkout'))

To delay JavaScript function call using jQuery

function sample() {
    alert("This is sample function");
}

$(function() {
    $("#button").click(function() {
        setTimeout(sample, 2000);
    });

});

jsFiddle.

If you want to encapsulate sample() there, wrap the whole thing in a self invoking function (function() { ... })().

Convert number of minutes into hours & minutes using PHP

$hours = floor($final_time_saving / 60);
$minutes = $final_time_saving % 60;

PHP expects T_PAAMAYIM_NEKUDOTAYIM?

This just happened to me in a foreach loop. I had inadvertently typed ($array as $key as $value) and PHP objected to the first as.

Determining if an Object is of primitive type

You have to deal with the auto-boxing of java.
Let's take the code

public class test
{
    public static void main(String [ ] args)
    {
        int i = 3;
        Object o = i;
        return;
    }
}
You get the class test.class and javap -c test let's you inspect the generated bytecode.
Compiled from "test.java"
public class test extends java.lang.Object{
public test();
  Code:
   0:   aload_0
   1:   invokespecial   #1; //Method java/lang/Object."":()V
   4:   return

public static void main(java.lang.String[]); Code: 0: iconst_3 1: istore_1 2: iload_1 3: invokestatic #2; //Method java/lang/Integer.valueOf:(I)Ljava/lang/Integer; 6: astore_2 7: return

}

As you can see the java compiler added
invokestatic    #2; //Method java/lang/Integer.valueOf:(I)Ljava/lang/Integer;
to create a new Integer from your int and then stores that new Object in o via astore_2

How do write IF ELSE statement in a MySQL query

You probably want to use a CASE expression.

They look like this:

SELECT col1, col2, (case when (action = 2 and state = 0) 
 THEN
      1 
 ELSE
      0 
 END)
 as state from tbl1;

error C2039: 'string' : is not a member of 'std', header file problem

Your FMAT.h requires a definition of std::string in order to complete the definition of class FMAT. In FMAT.cpp, you've done this by #include <string> before #include "FMAT.h". You haven't done that in your main file.

Your attempt to forward declare string was incorrect on two levels. First you need a fully qualified name, std::string. Second this works only for pointers and references, not for variables of the declared type; a forward declaration doesn't give the compiler enough information about what to embed in the class you're defining.

How to split the filename from a full path in batch?

Continuing from Pete's example above, to do it directly in the cmd window, use a single %, eg:

cd c:\test\folder A
for %X in (*)do echo %~nxX

(Note that special files like desktop.ini will not show up.)

It's also possible to redirect the output to a file using >>:

cd c:\test\folder A
for %X in (*)do echo %~nxX>>c:\test\output.txt

For a real example, assuming you want to robocopy all files from folder-A to folder-B (non-recursively):

cd c:\test\folder A
for %X in (*)do robocopy . "c:\test\folder B" "%~nxX" /dcopy:dat /copyall /v>>c:\test\output.txt

and for all folders (recursively):

cd c:\test\folder A
for /d %X in (*)do robocopy "%X" "C:\test\folder B\%X" /e /copyall /dcopy:dat /v>>c:\test\output2.txt

Undoing accidental git stash pop

If your merge was not too complicated another option would be to:

  1. Move all the changes including the merge changes back to stash using "git stash"
  2. Run the merge again and commit your changes (without the changes from the dropped stash)
  3. Run a "git stash pop" which should ignore all the changes from your previous merge since the files are identical now.

After that you are left with only the changes from the stash you dropped too early.

Javascript Array of Functions

It's basically the same as Darin Dimitrov's but it shows how you could use it do dynamically create and store functions and arguments. I hope it's useful for you :)

_x000D_
_x000D_
var argsContainer = ['hello', 'you', 'there'];_x000D_
var functionsContainer = [];_x000D_
_x000D_
for (var i = 0; i < argsContainer.length; i++) {_x000D_
var currentArg = argsContainer[i]; _x000D_
_x000D_
  functionsContainer.push(function(currentArg){_x000D_
    console.log(currentArg);_x000D_
  });_x000D_
};_x000D_
_x000D_
for (var i = 0; i < functionsContainer.length; i++) {_x000D_
  functionsContainer[i](argsContainer[i]);_x000D_
}
_x000D_
_x000D_
_x000D_

What is the connection string for localdb for version 11

I have connection string Server=(localdb)\v11.0;Integrated Security=true;Database=DB1;

and even a .NET 3.5 program connects and execute SQL successfully.

But many people say .NET 4.0.2 or 4.5 is required.

Tools to generate database tables diagram with Postgresql?

PostgreSQL Autodoc has worked well for me. It is a simple command line tool. From the web page:

This is a utility which will run through PostgreSQL system tables and returns HTML, Dot, Dia and DocBook XML which describes the database.

MVC which submit button has been pressed

you can identify your button from there name tag like below, You need to check like this in you controller

if (Request.Form["submit"] != null)
{
//Write your code here
}
else if (Request.Form["process"] != null)
{
//Write your code here
}

SQL Server 2008 Row Insert and Update timestamps

As an alternative to using a trigger, you might like to consider creating a stored procedure to handle the INSERTs that takes most of the columns as arguments and gets the CURRENT_TIMESTAMP which it includes in the final INSERT to the database. You could do the same for the CREATE. You may also be able to set things up so that users cannot execute INSERT and CREATE statements other than via the stored procedures.

I have to admit that I haven't actually done this myself so I'm not at all sure of the details.

"OverflowError: Python int too large to convert to C long" on windows but not mac

You'll get that error once your numbers are greater than sys.maxsize:

>>> p = [sys.maxsize]
>>> preds[0] = p
>>> p = [sys.maxsize+1]
>>> preds[0] = p
Traceback (most recent call last):
  File "<stdin>", line 1, in <module>
OverflowError: Python int too large to convert to C long

You can confirm this by checking:

>>> import sys
>>> sys.maxsize
2147483647

To take numbers with larger precision, don't pass an int type which uses a bounded C integer behind the scenes. Use the default float:

>>> preds = np.zeros((1, 3))

PHP: maximum execution time when importing .SQL data file

You're trying to import a huge dataset via a web interface.

By default PHP scripts run in the context of a web server have a maximum execution time limit because you don't want a single errant PHP script tying up the entire server and causing a denial of service.

For that reason your import is failing. PHPMyAdmin is a web application and is hitting the limit imposed by PHP.

You could try raising the limit but that limit exists for a good reason so that's not advisable. Running a script that is going to take a very long time to execute in a web server is a very bad idea.

PHPMyAdmin isn't really intended for heavy duty jobs like this, it's meant for day to day housekeeping tasks and troubleshooting.

Your best option is to use the proper tools for the job, such as the mysql commandline tools. Assuming your file is an SQL dump then you can try running the following from the commandline:

mysql -u(your user name here) -p(your password here) -h(your sql server name here) (db name here) < /path/to/your/sql/dump.sql

Or if you aren't comfortable with commandline tools then something like SQLYog (for Windows), Sequel Pro (for Mac), etc may be more suitable for running an import job

When should I use a trailing slash in my URL?

When you make your URL /about-us/ (with the trailing slash), it's easy to start with a single file index.html and then later expand it and add more files (e.g. our-CEO-john-doe.jpg) or even build a hierarchy under it (e.g. /about-us/company/, /about-us/products/, etc.) as needed, without changing the published URL. This gives you a great flexibility.

Spring Boot and how to configure connection details to MongoDB?

You can define more details by extending AbstractMongoConfiguration.

@Configuration
@EnableMongoRepositories("demo.mongo.model")
public class SpringMongoConfig extends AbstractMongoConfiguration {
    @Value("${spring.profiles.active}")
    private String profileActive;

    @Value("${spring.application.name}")
    private String proAppName;

    @Value("${spring.data.mongodb.host}")
    private String mongoHost;

    @Value("${spring.data.mongodb.port}")
    private String mongoPort;

    @Value("${spring.data.mongodb.database}")
    private String mongoDB;

    @Override
    public MongoMappingContext mongoMappingContext()
        throws ClassNotFoundException {
        // TODO Auto-generated method stub
        return super.mongoMappingContext();
    }
    @Override
    @Bean
    public Mongo mongo() throws Exception {
        return new MongoClient(mongoHost + ":" + mongoPort);
    }
    @Override
    protected String getDatabaseName() {
        // TODO Auto-generated method stub
        return mongoDB;
    }
}

Android How to adjust layout in Full Screen Mode when softkeyboard is visible

only use android:windowSoftInputMode="adjustResize|stateHidden as you use AdjustPan then it disable the resizing property

How to .gitignore all files/folder in a folder, but not the folder itself?

Put this .gitignore into the folder, then git add .gitignore.

*
*/
!.gitignore

The * line tells git to ignore all files in the folder, but !.gitignore tells git to still include the .gitignore file. This way, your local repository and any other clones of the repository all get both the empty folder and the .gitignore it needs.

Edit: May be obvious but also add */ to the .gitignore to also ignore subfolders.

Adding click event listener to elements with the same class

I find it more convenient to use something like the following:

document.querySelector('*').addEventListener('click',function(event){

    if( event.target.tagName != "IMG"){
        return;
    }

    // HANDLE CLICK ON IMAGES HERE
});

jQuery Validate Required Select

The solution mentioned by @JMP worked in my case with a little modification: I use element.value instead of value in the addmethod.

$.validator.addMethod("valueNotEquals", function(value, element, arg){
    // I use element.value instead value here, value parameter was always null
    return arg != element.value; 
}, "Value must not equal arg.");

// configure your validation
$("form").validate({
    rules: {
        SelectName: { valueNotEquals: "0" }
    },
    messages: {
        SelectName: { valueNotEquals: "Please select an item!" }
    }  
});

It could be possible, that I have a special case here, but didn't track down the cause. But @JMP's solution should work in regular cases.

Unable to compile class for JSP

From the error it seems that you are trying to import something which is not a class.

If your MyFunctions is a class, you should import it like this:

<%@page import="com.TransportPortal.MyFunctions"%>

If it is a package and you want to import everything in the package you should do like this:

<%@page import="com.TransportPortal.MyFunctions.* "%>

Edit:

There are two cases which will give you this error, edited to cover both.

How to replace plain URLs with links?

Here's my solution:

var content = "Visit https://wwww.google.com or watch this video: https://www.youtube.com/watch?v=0T4DQYgsazo and news at http://www.bbc.com";
content = replaceUrlsWithLinks(content, "http://");
content = replaceUrlsWithLinks(content, "https://");

function replaceUrlsWithLinks(content, protocol) {
    var startPos = 0;
    var s = 0;

    while (s < content.length) {
        startPos = content.indexOf(protocol, s);

        if (startPos < 0)
            return content;

        let endPos = content.indexOf(" ", startPos + 1);

        if (endPos < 0)
            endPos = content.length;

        let url = content.substr(startPos, endPos - startPos);

        if (url.endsWith(".") || url.endsWith("?") || url.endsWith(",")) {
            url = url.substr(0, url.length - 1);
            endPos--;
        }

        if (ROOTNS.utils.stringsHelper.validUrl(url)) {
            let link = "<a href='" + url + "'>" + url + "</a>";
            content = content.substr(0, startPos) + link + content.substr(endPos);
            s = startPos + link.length;
        } else {
            s = endPos + 1;
        }
    }

    return content;
}

function validUrl(url) {
    try {
        new URL(url);
        return true;
    } catch (e) {
        return false;
    }
}

How do you get the magnitude of a vector in Numpy?

Fastest way I found is via inner1d. Here's how it compares to other numpy methods:

import numpy as np
from numpy.core.umath_tests import inner1d

V = np.random.random_sample((10**6,3,)) # 1 million vectors
A = np.sqrt(np.einsum('...i,...i', V, V))
B = np.linalg.norm(V,axis=1)   
C = np.sqrt((V ** 2).sum(-1))
D = np.sqrt((V*V).sum(axis=1))
E = np.sqrt(inner1d(V,V))

print [np.allclose(E,x) for x in [A,B,C,D]] # [True, True, True, True]

import cProfile
cProfile.run("np.sqrt(np.einsum('...i,...i', V, V))") # 3 function calls in 0.013 seconds
cProfile.run('np.linalg.norm(V,axis=1)')              # 9 function calls in 0.029 seconds
cProfile.run('np.sqrt((V ** 2).sum(-1))')             # 5 function calls in 0.028 seconds
cProfile.run('np.sqrt((V*V).sum(axis=1))')            # 5 function calls in 0.027 seconds
cProfile.run('np.sqrt(inner1d(V,V))')                 # 2 function calls in 0.009 seconds

inner1d is ~3x faster than linalg.norm and a hair faster than einsum

What are the different types of keys in RDBMS?

(I) Super Key – An attribute or a combination of attribute that is used to identify the records uniquely is known as Super Key. A table can have many Super Keys.

E.g. of Super Key

  1. ID
  2. ID, Name
  3. ID, Address
  4. ID, Department_ID
  5. ID, Salary
  6. Name, Address
  7. Name, Address, Department_ID

So on as any combination which can identify the records uniquely will be a Super Key.

(II) Candidate Key – It can be defined as minimal Super Key or irreducible Super Key. In other words an attribute or a combination of attribute that identifies the record uniquely but none of its proper subsets can identify the records uniquely.

E.g. of Candidate Key

  1. ID
  2. Name, Address

For above table we have only two Candidate Keys (i.e. Irreducible Super Key) used to identify the records from the table uniquely. ID Key can identify the record uniquely and similarly combination of Name and Address can identify the record uniquely, but neither Name nor Address can be used to identify the records uniquely as it might be possible that we have two employees with similar name or two employees from the same house.

(III) Primary Key – A Candidate Key that is used by the database designer for unique identification of each row in a table is known as Primary Key. A Primary Key can consist of one or more attributes of a table.

E.g. of Primary Key - Database designer can use one of the Candidate Key as a Primary Key. In this case we have “ID” and “Name, Address” as Candidate Key, we will consider “ID” Key as a Primary Key as the other key is the combination of more than one attribute.

(IV) Foreign Key – A foreign key is an attribute or combination of attribute in one base table that points to the candidate key (generally it is the primary key) of another table. The purpose of the foreign key is to ensure referential integrity of the data i.e. only values that are supposed to appear in the database are permitted.

E.g. of Foreign Key – Let consider we have another table i.e. Department Table with Attributes “Department_ID”, “Department_Name”, “Manager_ID”, ”Location_ID” with Department_ID as an Primary Key. Now the Department_ID attribute of Employee Table (dependent or child table) can be defined as the Foreign Key as it can reference to the Department_ID attribute of the Departments table (the referenced or parent table), a Foreign Key value must match an existing value in the parent table or be NULL.

(V) Composite Key – If we use multiple attributes to create a Primary Key then that Primary Key is called Composite Key (also called a Compound Key or Concatenated Key).

E.g. of Composite Key, if we have used “Name, Address” as a Primary Key then it will be our Composite Key.

(VI) Alternate Key – Alternate Key can be any of the Candidate Keys except for the Primary Key.

E.g. of Alternate Key is “Name, Address” as it is the only other Candidate Key which is not a Primary Key.

(VII) Secondary Key – The attributes that are not even the Super Key but can be still used for identification of records (not unique) are known as Secondary Key.

E.g. of Secondary Key can be Name, Address, Salary, Department_ID etc. as they can identify the records but they might not be unique.

Set initially selected item in Select list in Angular2

Okay, so I figured out what the problem was, and the approach I believe works best. In my case, because the two objects weren't identical from a Javascript perspective, as in: they may have shared the same values, but they were different actual objects, e.g. originalObject was instantiated entirely separately from objects which was essentially an array of reference data (to populate the dropdown).

I found that the approach that worked best for me was to compare a unique property of the objects, rather than directly compare the two entire objects. This comparison is done in the bound property selected:

<select [ngModel]="originalObject">
    <option *ngFor="let object of objects" [ngValue]="object" [selected]="object.uniqueId === originalObject.uniqueId">{{object.name}}</option>
</select>

Calendar.getInstance(TimeZone.getTimeZone("UTC")) is not returning UTC time

You are definitely missing a small thing and that is you are not setting a default value:

TimeZone.setDefault(TimeZone.getTimeZone("UTC"));

So the code would look like:

TimeZone.setDefault(TimeZone.getTimeZone("UTC"));
Calendar cal_Two = Calendar.getInstance(TimeZone.getTimeZone("UTC"));
System.out.println(cal_Two.getTime());

Explanation: If you want to change the time zone, set the default time zone using TimeZone.setDefault()

Mongodb: failed to connect to server on first connect

I connected to a VPN and it worked. I was using school's WiFi.

CSS last-child(-1)

Unless you can get PHP to label that element with a class you are better to use jQuery.

jQuery(document).ready(function () {
  $count =  jQuery("ul li").size() - 1;
  alert($count);
  jQuery("ul li:nth-child("+$count+")").css("color","red");
});

Pass parameter to EventHandler

If I understand your problem correctly, you are calling a method instead of passing it as a parameter. Try the following:

myTimer.Elapsed += PlayMusicEvent;

where

public void PlayMusicEvent(object sender, ElapsedEventArgs e)
{
    music.player.Stop();
    System.Timers.Timer myTimer = (System.Timers.Timer)sender;
    myTimer.Stop();
}

But you need to think about where to store your note.

Deleting records before a certain date

To show result till yesterday

WHERE DATE(date_time) < CURDATE()

To show results of 10 days

WHERE date_time < NOW() - INTERVAL 10 DAY

To show results before 10 days

WHERE DATE(date_time) < DATE(NOW() - INTERVAL 10 DAY)

These will work for you

You can find dates like this

SELECT DATE(NOW() - INTERVAL 11 DAY)

Why my $.ajax showing "preflight is invalid redirect error"?

This answer goes over the exact same thing (although for angular) -- it is a CORS issue.

One quick fix is to modify each POST request by specifying one of the 'Content-Type' header values which will not trigger a "preflight". These types are:

  • application/x-www-form-urlencoded
  • multipart/form-data
  • text/plain

ANYTHING ELSE triggers a preflight.

For example:

$.ajax({
   url: 'http://api.example.com/users/get',
   type: 'POST',
   headers: {
      'name-api-key':'ewf45r4435trge',
      'Content-Type':'application/x-www-form-urlencoded'
   },
   data: {
      'uid':36,
   },
   success: function(data) {
      console.log(data);
   }
});

Python: how to print range a-z?

Try:

strng = ""
for i in range(97,123):
    strng = strng + chr(i)
print(strng)

How to use Apple's new San Francisco font on a webpage

None of the current answers including the accepted one will use Apple's San Francisco font on systems that don't have it installed as the system font. Since the question isn't "how do I use the OS X system font on a webpage" the correct solution is to use web fonts:

@font-face {
  font-family: "San Francisco";
  font-weight: 400;
  src: url("https://applesocial.s3.amazonaws.com/assets/styles/fonts/sanfrancisco/sanfranciscodisplay-regular-webfont.woff");
}

Source

How do I escape spaces in path for scp copy in Linux?

works

scp localhost:"f/a\ b\ c" .

scp localhost:'f/a\ b\ c' .

does not work

scp localhost:'f/a b c' .

The reason is that the string is interpreted by the shell before the path is passed to the scp command. So when it gets to the remote the remote is looking for a string with unescaped quotes and it fails

To see this in action, start a shell with the -vx options ie bash -vx and it will display the interpolated version of the command as it runs it.

C# removing items from listbox

You can't use an enumerator, you have to loop using an index, starting at the last item:

for (int n = listBox1.Items.Count - 1; n >= 0; --n)
{
    string removelistitem = "OBJECT";
    if (listBox1.Items[n].ToString().Contains(removelistitem))
    {
        listBox1.Items.RemoveAt(n);
    }
}

To find first N prime numbers in python

Try using while loop to check the count, that is easy. Find the code snippet below :

i=1
count = 0;
x = int(input("Enter the number:\n"))
while (count < x):
c=0
for j in range (1, (i+1), 1):
    a = i%j
    if (a==0):
        c = c+1

if (c==2):
      print (i)
      count = count+1
i=i+1

How to Insert BOOL Value to MySQL Database

TRUE and FALSE are keywords, and should not be quoted as strings:

INSERT INTO first VALUES (NULL, 'G22', TRUE);
INSERT INTO first VALUES (NULL, 'G23', FALSE);

By quoting them as strings, MySQL will then cast them to their integer equivalent (since booleans are really just a one-byte INT in MySQL), which translates into zero for any non-numeric string. Thus, you get 0 for both values in your table.

Non-numeric strings cast to zero:

mysql> SELECT CAST('TRUE' AS SIGNED), CAST('FALSE' AS SIGNED), CAST('12345' AS SIGNED);
+------------------------+-------------------------+-------------------------+
| CAST('TRUE' AS SIGNED) | CAST('FALSE' AS SIGNED) | CAST('12345' AS SIGNED) |
+------------------------+-------------------------+-------------------------+
|                      0 |                       0 |                   12345 |
+------------------------+-------------------------+-------------------------+

But the keywords return their corresponding INT representation:

mysql> SELECT TRUE, FALSE;
+------+-------+
| TRUE | FALSE |
+------+-------+
|    1 |     0 |
+------+-------+

Note also, that I have replaced your double-quotes with single quotes as are more standard SQL string enclosures. Finally, I have replaced your empty strings for id with NULL. The empty string may issue a warning.

How do I insert a JPEG image into a python Tkinter window?

import tkinter as tk
from tkinter import ttk
from PIL import Image,  ImageTk
win = tk. Tk()
image1 = Image. open("Aoran. jpg")
image2 =  ImageTk. PhotoImage(image1)
image_label = ttk. Label(win , image =.image2)
image_label.place(x = 0 , y = 0)
win.mainloop()

Ignore python multiple return value

If this is a function that you use all the time but always discard the second argument, I would argue that it is less messy to create an alias for the function without the second return value using lambda.

def func():
    return 1, 2

func_ = lambda: func()[0] 

func_()  # Prints 1 

How to allow Cross domain request in apache2

I had a lot of trouble getting this to work. Dummy me, don't forget that old page - even for sub-requests - gets cached in your browser. Maybe obvious, but clear your browsers cache. After that, one can also use Header set Cache-Control "no-store" This was helpful to me while testing.

Using msbuild to execute a File System Publish Profile

It looks to me like your publish profile is not being used, and doing some default packaging. The Microsoft Web Publish targets do all what you are doing above, it selects the correct targets based on the config.

I got mine to work no problem from TeamCity MSBuild step, but I did specify an explicit path to the profile, you just have to call it by name with no .pubxml (e.g. FileSystemDebug). It will be found so long as in the standard folder, which yours is.

Example:

C:\Windows\Microsoft.NET\Framework\v4.0.30319\MSBuild.exe ./ProjectRoot/MyProject.csproj /p:DeployOnBuild=true /p:PublishProfile=FileSystemDebug

Note this was done using the Visual Studio 2012 versions of the Microsoft Web Publish targets, normally located at "C:\Program Files (x86)\MSBuild\Microsoft\VisualStudio\v11.0\Web". Check out the deploy folder for the specific deployment types targets that are used

Align image in center and middle within div

    <div>
    <p style="text-align:center; margin-top:0px; margin-bottom:0px; padding:0px;">
    <img src="image.jpg" alt="image"/>
    </p>    
    </div>

How to change an Eclipse default project into a Java project

I deleted the project without removing content. I then created a new Java project from an existing resource. Pointing at my SVN checkout root folder. This worked for me. Although, Chris' way would have been much quicker. That's good to note for future. Thanks!

Android: How to programmatically access the device serial number shown in the AVD manager (API Version 8)

From Android P, defining the READ_PHONE_STATE permission in AndroidManifest only, will not work. We have to actually request for the permission. Below code works for me:

@RequiresApi(api = Build.VERSION_CODES.P)
@Override
protected void onCreate(Bundle savedInstanceState) {
    super.onCreate(savedInstanceState);
    setContentView(R.layout.activity_main);

    if (ActivityCompat.checkSelfPermission(this, Manifest.permission.READ_PHONE_STATE) != PackageManager.PERMISSION_GRANTED) {
        ActivityCompat.requestPermissions(this, new String[]{Manifest.permission.READ_PHONE_STATE}, 101);
    }
}

@RequiresApi(api = Build.VERSION_CODES.O)
@Override
protected void onResume() {
    super.onResume();
    if (ActivityCompat.checkSelfPermission(this, Manifest.permission.READ_PHONE_STATE) != PackageManager.PERMISSION_GRANTED) {
        return;
    }
    Log.d(TAG,Build.getSerial());
}
@RequiresApi(api = Build.VERSION_CODES.O)
@Override
public void onRequestPermissionsResult(int requestCode, String[] permissions, int[] grantResults) {
    switch (requestCode) {
        case 101:
            if (grantResults[0] == PackageManager.PERMISSION_GRANTED) {
            if (ActivityCompat.checkSelfPermission(this, Manifest.permission.READ_PHONE_STATE) != PackageManager.PERMISSION_GRANTED) {
                return;
            }
        } else {
            //not granted
        }
        break;
        default:
            super.onRequestPermissionsResult(requestCode, permissions, grantResults);
    }
}

Add this permissions in AndroidManifest.xml

<uses-permission android:name = "android.permission.INTERNET"/>
<uses-permission android:name = "android.permission.READ_PHONE_STATE" />

Hope this helps.

Thank You, MJ

Fixing "Lock wait timeout exceeded; try restarting transaction" for a 'stuck" Mysql table?

When you establish a connection for a transaction, you acquire a lock before performing the transaction. If not able to acquire the lock, then you try for sometime. If lock is still not obtainable, then lock wait time exceeded error is thrown. Why you will not able to acquire a lock is that you are not closing the connection. So, when you are trying to get a lock second time, you will not be able to acquire the lock as your previous connection is still unclosed and holding the lock.

Solution: close the connection or setAutoCommit(true) (according to your design) to release the lock.

ERROR 2003 (HY000): Can't connect to MySQL server (111)

Check that your remote host (i.e. the web hosting server you're trying to connect FROM) allows OUTGOING traffic on port 3306.

I saw the (100) error in this situation. I could connect from my PC/Mac, but not from my website. The MySQL instance was accessible via the internet, but my hosting company wasn't allowing my website to connect to the database on port 3306.

Once I asked my hosting company to open my web hosting account up to outgoing traffic on port 3306, my website could connect to my remote database.

How to use Javascript to read local text file and read line by line?

Using ES6 the javascript becomes a little cleaner

handleFiles(input) {

    const file = input.target.files[0];
    const reader = new FileReader();

    reader.onload = (event) => {
        const file = event.target.result;
        const allLines = file.split(/\r\n|\n/);
        // Reading line by line
        allLines.forEach((line) => {
            console.log(line);
        });
    };

    reader.onerror = (event) => {
        alert(event.target.error.name);
    };

    reader.readAsText(file);
}

Angular2 material dialog has issues - Did you add it to @NgModule.entryComponents?

If you're trying to use MatDialog inside a service - let's call it 'PopupService' and that service is defined in a module with:

@Injectable({ providedIn: 'root' })

then it may not work. I am using lazy loading, but not sure if that's relevant or not.

You have to:

  • Provide your PopupService directly to the component that opens your dialog - using [ provide: PopupService ]. This allows it to use (with DI) the MatDialog instance in the component. I think the component calling open needs to be in the same module as the dialog component in this instance.
  • Move the dialog component up to your app.module (as some other answers have said)
  • Pass a reference for matDialog when you call your service.

Excuse my jumbled answer, the point being it's the providedIn: 'root' that is breaking things because MatDialog needs to piggy-back off a component.

Aborting a shell script if any command returns a non-zero value

I am just throwing in another one for reference since there was an additional question to Mark Edgars input and here is an additional example and touches on the topic overall:

[[ `cmd` ]] && echo success_else_silence

Which is the same as cmd || exit errcode as someone showed.

For example, I want to make sure a partition is unmounted if mounted:

[[ `mount | grep /dev/sda1` ]] && umount /dev/sda1

Axios get in url works but with second parameter as object it doesn't

On client:

  axios.get('/api', {
      params: {
        foo: 'bar'
      }
    });

On server:

function get(req, res, next) {

  let param = req.query.foo
   .....
}

pip issue installing almost any library

For me, the latest pip (1.5.6) works fine with the insecure nltk package if you just tell it not to be so picky about security:

pip install --upgrade --force-reinstall --allow-all-external --allow-unverified ntlk nltk

How to get Text BOLD in Alert or Confirm box?

Maybe you coul'd use UTF8 bold chars.

For examples: https://yaytext.com/bold-italic/

It works on Chromium 80.0, I don't know on other browsers...

Allow only numbers to be typed in a textbox

You could subscribe for the onkeypress event:

<input type="text" class="textfield" value="" id="extra7" name="extra7" onkeypress="return isNumber(event)" />

and then define the isNumber function:

function isNumber(evt) {
    evt = (evt) ? evt : window.event;
    var charCode = (evt.which) ? evt.which : evt.keyCode;
    if (charCode > 31 && (charCode < 48 || charCode > 57)) {
        return false;
    }
    return true;
}

You can see it in action here.

'Access denied for user 'root'@'localhost' (using password: NO)'

For MySQL 5.7. These are the below steps:

Stop your MySQL server completely. This can be done by accessing the Services window inside Windows XP and Windows Server 2003, where you can stop the MySQL service.

Open your MS-DOS command prompt using "cmd" inside the Run window. Inside it navigate to your MySQL bin folder, such as C:\MySQL\bin using the cd command.

Execute the following command in the command prompt: mysqld.exe -u root --skip-grant-tables

Leave the current MS-DOS command prompt as it is, and open a new MS-DOS command prompt window.

Navigate to your MySQL bin folder, such as C:\MySQL\bin using the cd command.

Enter mysql and press enter.

You should now have the MySQL command prompt working. Type use mysql; so that we switch to the "mysql" database.

Execute the following command to update the password:

update user set authentication_string=password('1111') where user='root';

ggplot with 2 y axes on each side and different scales

You can create a scaling factor which is applied to the second geom and right y-axis. This is derived from Sebastian's solution.

library(ggplot2)

scaleFactor <- max(mtcars$cyl) / max(mtcars$hp)

ggplot(mtcars, aes(x=disp)) +
  geom_smooth(aes(y=cyl), method="loess", col="blue") +
  geom_smooth(aes(y=hp * scaleFactor), method="loess", col="red") +
  scale_y_continuous(name="cyl", sec.axis=sec_axis(~./scaleFactor, name="hp")) +
  theme(
    axis.title.y.left=element_text(color="blue"),
    axis.text.y.left=element_text(color="blue"),
    axis.title.y.right=element_text(color="red"),
    axis.text.y.right=element_text(color="red")
  )

enter image description here

Note: using ggplot2 v3.0.0

CAST to DECIMAL in MySQL

From MySQL docs: Fixed-Point Types (Exact Value) - DECIMAL, NUMERIC:

In standard SQL, the syntax DECIMAL(M) is equivalent to DECIMAL(M,0)

So, you are converting to a number with 2 integer digits and 0 decimal digits. Try this instead:

CAST((COUNT(*) * 1.5) AS DECIMAL(12,2)) 

Is there a way to avoid null check before the for-each loop iteration starts?

If possible, you should design your code such that the collections aren't null in the first place.

null collections are bad practice (for this reason); you should use empty collections instead. (eg, Collections.emptyList())

Alternatively, you could make a wrapper class that implements Iterable and takes a collections, and handles a null collection.
You could then write foreach(T obj : new Nullable<T>(list1))

Summing elements in a list

Python iterable can be summed like so - [sum(range(10)[1:])] . This sums all elements from the list except the first element.

>>> atuple = (1,2,3,4,5)
>>> sum(atuple)
15
>>> alist = [1,2,3,4,5]
>>> sum(alist)
15

The network adapter could not establish the connection - Oracle 11g

First check your listener is on or off. Go to net manager then Local -> service naming -> orcl. Then change your HOST NAME and put your PC name. Now go to LISTENER and change the HOST and put your PC name.

Cannot deserialize instance of object out of START_ARRAY token in Spring Webservice

I've had a very similar issue using spring-boot-starter-data-redis. To my implementation there was offered a @Bean for RedisTemplate as follows:

@Bean
public RedisTemplate<String, List<RoutePlantCache>> redisTemplate(RedisConnectionFactory connectionFactory) {
    final RedisTemplate<String, List<RoutePlantCache>> template = new RedisTemplate<>();
    template.setConnectionFactory(connectionFactory);
    template.setKeySerializer(new StringRedisSerializer());
    template.setValueSerializer(new Jackson2JsonRedisSerializer<>(RoutePlantCache.class));

    // Add some specific configuration here. Key serializers, etc.
    return template;
}

The fix was to specify an array of RoutePlantCache as following:

template.setValueSerializer(new Jackson2JsonRedisSerializer<>(RoutePlantCache[].class));

Below the exception I had:

com.fasterxml.jackson.databind.exc.MismatchedInputException: Cannot deserialize instance of `[...].RoutePlantCache` out of START_ARRAY token
 at [Source: (byte[])"[{ ... },{ ... [truncated 1478 bytes]; line: 1, column: 1]
    at com.fasterxml.jackson.databind.exc.MismatchedInputException.from(MismatchedInputException.java:59) ~[jackson-databind-2.11.4.jar:2.11.4]
    at com.fasterxml.jackson.databind.DeserializationContext.reportInputMismatch(DeserializationContext.java:1468) ~[jackson-databind-2.11.4.jar:2.11.4]
    at com.fasterxml.jackson.databind.DeserializationContext.handleUnexpectedToken(DeserializationContext.java:1242) ~[jackson-databind-2.11.4.jar:2.11.4]
    at com.fasterxml.jackson.databind.DeserializationContext.handleUnexpectedToken(DeserializationContext.java:1190) ~[jackson-databind-2.11.4.jar:2.11.4]
    at com.fasterxml.jackson.databind.deser.BeanDeserializer._deserializeFromArray(BeanDeserializer.java:604) ~[jackson-databind-2.11.4.jar:2.11.4]
    at com.fasterxml.jackson.databind.deser.BeanDeserializer._deserializeOther(BeanDeserializer.java:190) ~[jackson-databind-2.11.4.jar:2.11.4]
    at com.fasterxml.jackson.databind.deser.BeanDeserializer.deserialize(BeanDeserializer.java:166) ~[jackson-databind-2.11.4.jar:2.11.4]
    at com.fasterxml.jackson.databind.ObjectMapper._readMapAndClose(ObjectMapper.java:4526) ~[jackson-databind-2.11.4.jar:2.11.4]
    at com.fasterxml.jackson.databind.ObjectMapper.readValue(ObjectMapper.java:3572) ~[jackson-databind-2.11.4.jar:2.11.4]

Init function in javascript and how it works

That pattern will create a new execution context (EC) in which any local variable objects (VO's) will live, and will likewise die when the EC exits. The only exception to this lifetime is for VO's which become part of a closure.

Please note that JavaScript has no magic "init" function. You might associate this pattern with such since most any self-respecting JS library (jQuery, YUI, etc.) will do this so that they don't pollute the global NS more than they need to.

A demonstration:

var x = 1; // global VO
(function(){        
    var x = 2; // local VO
})();
x == 1; // global VO, unchanged by the local VO

The 2nd set of "brackets" (those are actually called parens, or a set of parentheses), are simply to invoke the function expression directly preceding it (as defined by the prior set of parenthesis).

rails generate model

The error shows you either didn't create the rails project yet or you're not in the rails project directory.

Suppose if you're working on myapp project. You've to move to that project directory on your command line and then generate the model. Here are some steps you can refer.

Example: Assuming you didn't create the Rails app yet:

$> rails new myapp
$> cd myapp

Now generate the model from your commandline.

$> rails generate model your_model_name 

Convert an array to string

You probably want something like this overload of String.Join:

String.Join<T> Method (String, IEnumerable<T>)

Docs:

http://msdn.microsoft.com/en-us/library/dd992421.aspx

In your example, you'd use

String.Join("", Client);

EditText non editable

android:editable="false" should work, but it is deprecated, you should be using android:inputType="none" instead.

Alternatively, if you want to do it in the code you could do this :

EditText mEdit = (EditText) findViewById(R.id.yourid);
mEdit.setEnabled(false);

This is also a viable alternative :

EditText mEdit = (EditText) findViewById(R.id.yourid);
mEdit.setKeyListener(null);

If you're going to make your EditText non-editable, may I suggest using the TextView widget instead of the EditText, since using a EditText seems kind of pointless in that case.

EDIT: Altered some information since I've found that android:editable is deprecated, and you should use android:inputType="none", but there is a bug about it on android code; So please check this.

Working with $scope.$emit and $scope.$on

Below code shows the two sub-controllers from where the events are dispatched upwards to parent controller (rootScope)

<body ng-app="App">

    <div ng-controller="parentCtrl">

        <p>City : {{city}} </p>
        <p> Address : {{address}} </p>

        <div ng-controller="subCtrlOne">
            <input type="text" ng-model="city" />
            <button ng-click="getCity(city)">City !!!</button>
        </div>

        <div ng-controller="subCtrlTwo">

            <input type="text" ng-model="address" />
            <button ng-click="getAddrress(address)">Address !!!</button>

        </div>

    </div>

</body>

var App = angular.module('App', []);

// parent controller
App.controller('parentCtrl', parentCtrl);

parentCtrl.$inject = ["$scope"];

function parentCtrl($scope) {

    $scope.$on('cityBoom', function(events, data) {
        $scope.city = data;
    });

    $scope.$on('addrBoom', function(events, data) {
        $scope.address = data;
    });
}

// sub controller one

App.controller('subCtrlOne', subCtrlOne);

subCtrlOne.$inject = ['$scope'];

function subCtrlOne($scope) {

    $scope.getCity = function(city) {

        $scope.$emit('cityBoom', city);    
    }
}

// sub controller two

App.controller('subCtrlTwo', subCtrlTwo);

subCtrlTwo.$inject = ["$scope"];

function subCtrlTwo($scope) {

    $scope.getAddrress = function(addr) {

        $scope.$emit('addrBoom', addr);   
    }
}

http://jsfiddle.net/shushanthp/zp6v0rut/

Android image caching

Late answer, but I figured I should add a link to my site because I have written a tutorial how to make an image cache for android: http://squarewolf.nl/2010/11/android-image-cache/ Update: the page has been taken offline as the source was outdated. I join @elenasys in her advice to use Ignition.

So to all the people who stumble upon this question and haven't found a solution: hope you enjoy! =D

Gridview with two columns and auto resized images

Here's a relatively easy method to do this. Throw a GridView into your layout, setting the stretch mode to stretch the column widths, set the spacing to 0 (or whatever you want), and set the number of columns to 2:

res/layout/main.xml

<?xml version="1.0" encoding="utf-8"?>
<FrameLayout 
    xmlns:android="http://schemas.android.com/apk/res/android"
    android:layout_width="match_parent"
    android:layout_height="match_parent">

    <GridView
        android:id="@+id/gridview"
        android:layout_width="match_parent"
        android:layout_height="match_parent"
        android:verticalSpacing="0dp"
        android:horizontalSpacing="0dp"
        android:stretchMode="columnWidth"
        android:numColumns="2"/>

</FrameLayout>

Make a custom ImageView that maintains its aspect ratio:

src/com/example/graphicstest/SquareImageView.java

public class SquareImageView extends ImageView {
    public SquareImageView(Context context) {
        super(context);
    }

    public SquareImageView(Context context, AttributeSet attrs) {
        super(context, attrs);
    }

    public SquareImageView(Context context, AttributeSet attrs, int defStyle) {
        super(context, attrs, defStyle);
    }

    @Override
    protected void onMeasure(int widthMeasureSpec, int heightMeasureSpec) {
        super.onMeasure(widthMeasureSpec, heightMeasureSpec);
        setMeasuredDimension(getMeasuredWidth(), getMeasuredWidth()); //Snap to width
    }
}

Make a layout for a grid item using this SquareImageView and set the scaleType to centerCrop:

res/layout/grid_item.xml

<?xml version="1.0" encoding="utf-8"?>

<FrameLayout xmlns:android="http://schemas.android.com/apk/res/android"
             android:layout_width="match_parent"
             android:layout_height="match_parent">

    <com.example.graphicstest.SquareImageView
        android:id="@+id/picture"
        android:layout_width="match_parent"
        android:layout_height="match_parent"
        android:scaleType="centerCrop"/>

    <TextView
        android:id="@+id/text"
        android:layout_width="match_parent"
        android:layout_height="wrap_content"
        android:paddingLeft="10dp"
        android:paddingRight="10dp"
        android:paddingTop="15dp"
        android:paddingBottom="15dp"
        android:layout_gravity="bottom"
        android:textColor="@android:color/white"
        android:background="#55000000"/>

</FrameLayout>

Now make some sort of adapter for your GridView:

src/com/example/graphicstest/MyAdapter.java

private final class MyAdapter extends BaseAdapter {
    private final List<Item> mItems = new ArrayList<Item>();
    private final LayoutInflater mInflater;

    public MyAdapter(Context context) {
        mInflater = LayoutInflater.from(context);

        mItems.add(new Item("Red",       R.drawable.red));
        mItems.add(new Item("Magenta",   R.drawable.magenta));
        mItems.add(new Item("Dark Gray", R.drawable.dark_gray));
        mItems.add(new Item("Gray",      R.drawable.gray));
        mItems.add(new Item("Green",     R.drawable.green));
        mItems.add(new Item("Cyan",      R.drawable.cyan));
    }

    @Override
    public int getCount() {
        return mItems.size();
    }

    @Override
    public Item getItem(int i) {
        return mItems.get(i);
    }

    @Override
    public long getItemId(int i) {
        return mItems.get(i).drawableId;
    }

    @Override
    public View getView(int i, View view, ViewGroup viewGroup) {
        View v = view;
        ImageView picture;
        TextView name;

        if (v == null) {
            v = mInflater.inflate(R.layout.grid_item, viewGroup, false);
            v.setTag(R.id.picture, v.findViewById(R.id.picture));
            v.setTag(R.id.text, v.findViewById(R.id.text));
        }

        picture = (ImageView) v.getTag(R.id.picture);
        name = (TextView) v.getTag(R.id.text);

        Item item = getItem(i);

        picture.setImageResource(item.drawableId);
        name.setText(item.name);

        return v;
    }

    private static class Item {
        public final String name;
        public final int drawableId;

        Item(String name, int drawableId) {
            this.name = name;
            this.drawableId = drawableId;
        }
    }
}

Set that adapter to your GridView:

@Override
public void onCreate(Bundle savedInstanceState) {
    super.onCreate(savedInstanceState);
    setContentView(R.layout.main);
    GridView gridView = (GridView)findViewById(R.id.gridview);
    gridView.setAdapter(new MyAdapter(this));
}

And enjoy the results:

Example GridView

navigator.geolocation.getCurrentPosition sometimes works sometimes doesn't

I'm still getting spotty results in 2017, and I have a theory: the API documentation says that the call is now only available "in a secure context", i.e. over HTTPS. I'm having trouble getting a result in my development environment (http on localhost) and I believe this is why.

Change directory in Node.js command prompt

Add the \d [dir] attribute to the cd command like this:

cd \d %yourdir%:\

Scala: what is the best way to append an element to an Array?

You can use :+ to append element to array and +: to prepend it:

0 +: array :+ 4

should produce:

res3: Array[Int] = Array(0, 1, 2, 3, 4)

It's the same as with any other implementation of Seq.

Which "href" value should I use for JavaScript links, "#" or "javascript:void(0)"?

Unless you're writing out the link using JavaScript (so that you know it's enabled in the browser), you should ideally be providing a proper link for people who are browsing with JavaScript disabled and then prevent the default action of the link in your onclick event handler. This way those with JavaScript enabled will run the function and those with JavaScript disabled will jump to an appropriate page (or location within the same page) rather than just clicking on the link and having nothing happen.

Execute write on doc: It isn't possible to write into a document from an asynchronously-loaded external script unless it is explicitly opened.

In case this is useful to anyone I had this same issue. I was bringing in a footer into a web page via jQuery. Inside that footer were some Google scripts for ads and retargeting. I had to move those scripts from the footer and place them directly in the page and that eliminated the notice.

How to iterate for loop in reverse order in swift?

Xcode 6 beta 4 added two functions to iterate on ranges with a step other than one: stride(from: to: by:), which is used with exclusive ranges and stride(from: through: by:), which is used with inclusive ranges.

To iterate on a range in reverse order, they can be used as below:

for index in stride(from: 5, to: 1, by: -1) {
    print(index)
}
//prints 5, 4, 3, 2

for index in stride(from: 5, through: 1, by: -1) {
    print(index)
}
//prints 5, 4, 3, 2, 1

Note that neither of those is a Range member function. They are global functions that return either a StrideTo or a StrideThrough struct, which are defined differently from the Range struct.

A previous version of this answer used the by() member function of the Range struct, which was removed in beta 4. If you want to see how that worked, check the edit history.

How to display custom view in ActionBar?

There is a trick for this. All you have to do is to use RelativeLayout instead of LinearLayout as the main container. It's important to have android:layout_gravity="fill_horizontal" set for it. That should do it.

Create a list with initial capacity in Python

As others have mentioned, the simplest way to preseed a list is with NoneType objects.

That being said, you should understand the way Python lists actually work before deciding this is necessary.

In the CPython implementation of a list, the underlying array is always created with overhead room, in progressively larger sizes ( 4, 8, 16, 25, 35, 46, 58, 72, 88, 106, 126, 148, 173, 201, 233, 269, 309, 354, 405, 462, 526, 598, 679, 771, 874, 990, 1120, etc), so that resizing the list does not happen nearly so often.

Because of this behavior, most list.append() functions are O(1) complexity for appends, only having increased complexity when crossing one of these boundaries, at which point the complexity will be O(n). This behavior is what leads to the minimal increase in execution time in S.Lott's answer.

Source: Python list implementation

Can not find the tag library descriptor of springframework

If you are using maven use this dependency:

<dependency>
    <groupId>org.springframework.security</groupId>
    <artifactId>spring-security-taglibs</artifactId>
    <version>3.1.4.RELEASE</version>
</dependency>

"Port 4200 is already in use" when running the ng serve command

ng serve --port 122345 is worked fine for me.

mvn command not found in OSX Mavrerick

Try following these if these might help:

Since your installation works on the terminal you installed, all the exports you did, work on the current bash and its child process. but is not spawned to new terminals.

env variables are lost if the session is closed; using .bash_profile, you can make it available in all sessions, since when a bash session starts, it 'runs' its .bashrc and .bash_profile

Now follow these steps and see if it helps:

  1. type env | grep M2_HOME on the terminal that is working. This should give something like

    M2_HOME=/usr/local/apache-maven/apache-maven-3.1.1

  2. typing env | grep JAVA_HOME should give like this:

    JAVA_HOME=/Library/Java/JavaVirtualMachines/jdk1.7.0_40.jdk/Contents/Home

Now you have the PATH for M2_HOME and JAVA_HOME.

If you just do ls /usr/local/apache-maven/apache-maven-3.1.1/bin, you will see mvn binary there. All you have to do now is to point to this location everytime using PATH. since bash searches in all the directory path mentioned in PATH, it will find mvn.

  1. now open .bash_profile, if you dont have one just create one

    vi ~/.bash_profile

Add the following:

#set JAVA_HOME
JAVA_HOME=/Library/Java/JavaVirtualMachines/jdk1.7.0_40.jdk/Contents/Home
export JAVA_HOME


M2_HOME=/usr/local/apache-maven/apache-maven-3.1.1
export M2_HOME

PATH=$PATH:$JAVA_HOME/bin:$M2_HOME/bin
export PATH
  1. save the file and type source ~/.bash_profile. This steps executes the commands in the .bash_profile file and you are good to go now.

  2. open a new terminal and type mvn that should work.

Chaining Observables in RxJS

About promise composition vs. Rxjs, as this is a frequently asked question, you can refer to a number of previously asked questions on SO, among which :

Basically, flatMap is the equivalent of Promise.then.

For your second question, do you want to replay values already emitted, or do you want to process new values as they arrive? In the first case, check the publishReplay operator. In the second case, standard subscription is enough. However you might need to be aware of the cold. vs. hot dichotomy depending on your source (cf. Hot and Cold observables : are there 'hot' and 'cold' operators? for an illustrated explanation of the concept)

Switch case with fallthrough?

Try this:

case $VAR in
normal)
    echo "This doesn't do fallthrough"
    ;;
special)
    echo -n "This does "
    ;&
fallthrough)
    echo "fall-through"
    ;;
esac

How to declare std::unique_ptr and what is the use of it?

The constructor of unique_ptr<T> accepts a raw pointer to an object of type T (so, it accepts a T*).

In the first example:

unique_ptr<int> uptr (new int(3));

The pointer is the result of a new expression, while in the second example:

unique_ptr<double> uptr2 (pd);

The pointer is stored in the pd variable.

Conceptually, nothing changes (you are constructing a unique_ptr from a raw pointer), but the second approach is potentially more dangerous, since it would allow you, for instance, to do:

unique_ptr<double> uptr2 (pd);
// ...
unique_ptr<double> uptr3 (pd);

Thus having two unique pointers that effectively encapsulate the same object (thus violating the semantics of a unique pointer).

This is why the first form for creating a unique pointer is better, when possible. Notice, that in C++14 we will be able to do:

unique_ptr<int> p = make_unique<int>(42);

Which is both clearer and safer. Now concerning this doubt of yours:

What is also not clear to me, is how pointers, declared in this way will be different from the pointers declared in a "normal" way.

Smart pointers are supposed to model object ownership, and automatically take care of destroying the pointed object when the last (smart, owning) pointer to that object falls out of scope.

This way you do not have to remember doing delete on objects allocated dynamically - the destructor of the smart pointer will do that for you - nor to worry about whether you won't dereference a (dangling) pointer to an object that has been destroyed already:

{
    unique_ptr<int> p = make_unique<int>(42);
    // Going out of scope...
}
// I did not leak my integer here! The destructor of unique_ptr called delete

Now unique_ptr is a smart pointer that models unique ownership, meaning that at any time in your program there shall be only one (owning) pointer to the pointed object - that's why unique_ptr is non-copyable.

As long as you use smart pointers in a way that does not break the implicit contract they require you to comply with, you will have the guarantee that no memory will be leaked, and the proper ownership policy for your object will be enforced. Raw pointers do not give you this guarantee.

Visual Studio Code: Auto-refresh file changes

{
    "files.useExperimentalFileWatcher" : true
}

in Code -> Preferences -> Settings

Tested with Visual Studio Code Version 1.26.1 on mac and win

Change Screen Orientation programmatically using a Button

Wherever possible, please don't use SCREEN_ORIENTATION_LANDSCAPE or SCREEN_ORIENTATION_PORTRAIT. Instead use:

setRequestedOrientation(ActivityInfo.SCREEN_ORIENTATION_SENSOR_LANDSCAPE);
setRequestedOrientation(ActivityInfo.SCREEN_ORIENTATION_SENSOR_PORTRAIT);

These allow the user to orient the device to either landscape orientation, or either portrait orientation, respectively. If you've ever had to play a game with a charging cable being driven into your stomach, then you know exactly why having both orientations available is important to the user.

Note: For phones, at least several that I've checked, it only allows the "right side up" portrait mode, however, SENSOR_PORTRAIT works properly on tablets.

Note: this feature was introduced in API Level 9, so if you must support 8 or lower (not likely at this point), then instead use:

setRequestedOrientation(Build.VERSION.SDK_INT < 9 ?
                        ActivityInfo.SCREEN_ORIENTATION_LANDSCAPE :
                        ActivityInfo.SCREEN_ORIENTATION_SENSOR_LANDSCAPE);
setRequestedOrientation(Build.VERSION.SDK_INT < 9 ?
                        ActivityInfo.SCREEN_ORIENTATION_PORTRAIT :
                        ActivityInfo.SCREEN_ORIENTATION_SENSOR_PORTRAIT);

Java system properties and environment variables

MySQL convert date string to Unix timestamp

You will certainly have to use both STR_TO_DATE to convert your date to a MySQL standard date format, and UNIX_TIMESTAMP to get the timestamp from it.

Given the format of your date, something like

UNIX_TIMESTAMP(STR_TO_DATE(Sales.SalesDate, '%M %e %Y %h:%i%p'))

Will gives you a valid timestamp. Look the STR_TO_DATE documentation to have more information on the format string.

Assigning default values to shell variables with a single command in bash

Here is an example

#!/bin/bash

default='default_value'
value=${1:-$default}

echo "value: [$value]"

save this as script.sh and make it executable. run it without params

./script.sh
> value: [default_value]

run it with param

./script.sh my_value
> value: [my_value]